MS spring

Ace your homework & exams now with Quizwiz!

The nurse is instructing a patient with heart failure about a prescribed sodium-restricted diet. Which patient statement indicates that additional teaching is required?

"*I can use as much salt substitute as I want*." Low-sodium salt substitutes are not really sodium-free. They may contain half as much sodium as regular salt. The patient should be instructed to use salt substitutes sparingly because larger amounts often taste bitter instead of salty.

The nurse is caring for a patient with a *potassium level of 5.9 mEq/L*. The health-care provider prescribes both glucose and insulin for the patient. The patient's spouse asks, "Why is insulin needed?" Which response by the nurse is the most appropriate?

"*The insulin will cause his extra potassium to move into his cells*, which will lower potassium in the blood." Serum potassium levels may be temporarily lowered by administering glucose and insulin, which cause potassium to leave the extracellular fluid and enter cells.

The nurse is caring for a patient who is undergoing diagnostic tests to rule out lung cancer. The patient asks the nurse why a computed tomography (CT) scan was ordered. What is the best response by the nurse?

"It is more specific in diagnosing your condition."

The nurse is teaching a patient scheduled for a colonoscopy on pre- and postprocedure care. Which statement by the patient indicates the need for further teaching?

"It might be quite painful."

The nurse is providing care to a pediatric patient who is HIV-positive. The patient's mother is describing the child's current condition and activities to the nurse. Which parental statement indicates that the child may require further intervention?

"My child seems somewhat isolated and doesn't have any real friends."

The nurse is providing care to a patient who experienced an allergic reaction. Which leukocyte does the nurse anticipate will be elevated?

*Basophils.* Monocytes are produced for phagocytosis to digest engulfed microorganisms. Elevated Eosinophils = parasitic infection Elevated neutrophils = bacterial infection

Which *chronic lung condition noted in the patient's health history supports* the current diagnosis of *respiratory acidosis*?

*Cystic fibrosis*

The nurse is caring for a patient admitted with *renal failure & metabolic acidosis*. Which clinical manifestation would indicate to the nurse that planned interventions to *relieve the metabolic acidosis have been effective*?

*Decreased depth of respirations* pt with metabolic acidosis will have increased respiratory rate and depth. effective care will decrease this

The nurse is caring for a *comatose patient with respiratory acidosis*. For which intervention will the nurse need to collaborate when caring for this patient?

*Determining recent eating behaviors*

The nurse is providing care to a patient with a compromised immune system. Which independent nursing intervention is appropriate for the nurse to include in the patient's plan of care?

*Educating on the importance of a nutritious diet.*

The nurse is caring for a patient admitted with *HTN* and *chronic renal failure* who receives hemodialysis three times per week. The nurse is assessing the patient's diet and notes the *use of salt substitutes*. When teaching the patient to avoid salt substitute, which rationale supports this teaching point?

*They can potentiate hyperkalemia*. Many salt substitutes use potassium chloride. Potassium intake is carefully regulated in patients with renal failure, and the use of salt substitutes will worsen hyperkalemia.

Euvolemic causes

- hypothyroidism -SIADH -Meds: carbamazepine, opioids, barbs, vincritine, phenytoin, pantoprazole

The nurse instructs a group of community members about ways to reduce the development of cancer. Which participant statements indicate that teaching has been effective? Select all that apply

-"I need to get my home tested for radon." -"I need to keep my children away from smokers." -'Sunscreen should be applied before spending time outdoors."

The nurse is providing discharge instructions to a patient being treated for cancer. For which symptoms should the patient be instructed to call for help at home? Select all that apply

-Desire to end life -Difficulty breathing -New onset of bleeding -Significant increase in vomiting

Hypophosphatemia treatment

-Phosphorous supplementation -Neutra-phos powder -KPhos IV replacement

Sodium content of common foods

-frozen pizza, plain cheese -tomato soup/juice -pretzels -potato chips -salsa -salad dressing -tortilla chips -breads -frozen vegetable

Which IV fluid should the nurse prepare when a pt requires an isotonic solution? 1. 0.9% NS 2. 2.5% 3. 0.33% sodium chloride 4. 5% dextrose in Lactated ringers

1. 0.9% NS

A pt reports a change in the taste of food. Which cranial nerve should the nurse suspect as being affected? 1. CN VII Facial 2. CN V Trigeminal 3. CN XI Accessory 4. CN XII Hypoglossal

1. CN VII Facial

Which component should the nurse anticipate will be prescribed for a pt w/ severe thrombocytopenia? 1. Platelets 2. Albumin 3. Fresh frozen plasma 4. Packed RBCs

1. Platelets

During a neurologic assessment the nurse asks a pt to close the eyes and asks the pt to identify a paper clip placed in the hand. What is the nurse assessing? 1. Stereognosis 2. Hyperesthesia 3. Graphesthesia 4. Two-point discrimination

1. Stereognosis

The nurse is caring for a pt w/ myasthenia gravis. Which assessment should the nurse complete to determine respiratory functioning? 1. Vital capacity 2. Pulse oximetry 3. Auscultate lung sounds 4. ABG analysis

1. Vital capacity

A patient's serum sodium level is *150 mg/dL*. Based on this data, which interventions should the nurse plan for this patient? (SATA)

1.) Instruct on a *low-sodium diet*. -For an elevated sodium level, the electrolyte will need to be restricted, in the form of a low-sodium diet. 2.) Administer *diuretics* as prescribed. -Diuretics will remove excess fluid being held in the body because of the extra sodium.

Which component should the nurse anticipate will be prescribed for a pt is not responding to crystalloids for volume expansion? 1. Platelets 2. Albumin 3. Fresh frozen plasma 4. Packed RBCs

2. Albumin

A pt w/ myasthenia gravis is experiencing sweating and pallor. After administering edrophonium (Tensilon), which finding suggests the pt is experiencing a cholinergic crisis? 1. Clear vision 2. Fasciculations 3. Strong hand grasps 4. Equal shoulder shrugs

2. Fasciculations

A pt is experiencing changes in eye movements. Which cell structure is being affected? 1. Pons 2. Midbrain 3. Medulla oblongata 4. Reticular formation

2. Midbrain

output

2500 mL/day feces= 100 mL breathing= 400 mL skin= 500mL urine= 1500 mL

It is believed that a pt's migraine headaches are caused by vascular constriction. Which medication should the nurse expect to be prescribed for this pt? 1. Citalopram (Celexa) 2. Imipramine (Tofranil) 3. Amlodipine (Norvsac) 4. Lamotrigine (Lamictal)

3. Amlodipine (Norvsac)

The nurse is reviewing orders written for a pt w/ trigeminal neuralgia. Which medication should the nurse expect to be prescribed for this pt? 1. Cogentin 2. Compazine 3. Carbamazepine 4. Hydrochlorothiazide

3. Carbamazepine

A pt is diagnosed with a health problem that causes demyelization of the peripheral nerves. Which cell structure is being affected? 1. Microglia 2. Astrocytes 3. Schwann cells 4. Oligodendrocytes

3. Schwann cells

A pt w/ trigeminal neuralgia asks if ther are any nonpharmacological strategies to treat the pain. What should the nurse suggest to this pt? 1. Massage 2. Apply heat 3. Apply cold 4. Acupuncture

4. Acupuncture

A pt is suspected of having Alzheimer's disease. What diagnostic testing should the nurse expect to provide a presumptive diagnosis of this disorder? 1. Echocardiogram 2. Cerebral angiogram 3. Serum medication levels 4. Hx & physical examination

4. Hx & physical examination

The nurse is providing care to a patient following hemodialysis. The patient is experiencing *tachycardia* and *decreased urine output* along with *skin* that is *pale and cool to the touch*. Which *goal of hemodialysis* does the nurse determine the patient has *not* met based on the current data?

A *reduction* of *extracellular fluid* The patient receiving hemodialysis is expected to have a reduction of extracellular fluid, not a fluid deficit that puts the patient at risk.

The nurse receives shift report on a pediatric medical-surgical unit. The nurse has been assigned four patients for the shift. Which child does the nurse plan to *assess first* based on the increased risk for dehydration?

A 15-month-old child with *tachypnea* The pediatric patient with the greatest risk for dehydration is the child who is under 2 years of age experiencing tachypnea which increases insensible fluid loss.

The nurse is caring for a patient in an allergy clinic. After completing the patient history, the nurse selects the nursing diagnosis of Risk for Shock. Which item in the patient's history supports the need for this nursing diagnosis?

A history of an anaphylactic reaction to shellfish.

16. A patient with a traumatic brain injury is leaking clear fluid from the nose. What action should the nurse take? 1) Collect the fluid with gauze 2) Check the fluid for red blood cells 3) Send a specimen for a protein level 4) Insert a nasal plug in the nostril leaking the fluid

ANS: 1 Feedback 1 If clear fluid is draining from the ear or nose, it should be collected using loosely applied gauze. 2 Clear drainage would be checked for glucose and not red blood cells. 3 Clear drainage would be checked for glucose and not protein. 4 If clear fluid is draining from the ear or nose, it should not be stopped.

A patient with multiple sclerosis is admitted for treatment of clinical manifestations. What should the nurse expect to be prescribed for this patient? 1) Corticosteroids 2) Beta interferons 3) Muscle relaxants 4) Immunosuppressive agents

ANS: 3 1 Corticosteroids are used to treat attacks. 2 Beta interferons are used to modify the disease course. 3 Medications used to treat clinical manifestations include muscle relaxants. 4 Immunosuppressive agents are used to modify the disease course.

6. The nurse is concerned that a patient is at high risk for having a stroke. What finding did the nurse use to make this clinical decision? 1) BMI 24.8 2) Heart rate 90 bpm 3) Blood pressure 182/90 mm Hg 4) Pulse oximetry 98% on room air

ANS: 3 Feedback 1 Overweight is not identified as a risk factor for stroke. 2 Heart rate of 90 bpm is within normal limits and is not a risk factor for stroke. 3 A significant risk factor for stroke is hypertension. 4 A pulse oximeter reading of 98% on room air indicates adequate oxygenation and is not a risk factor for stroke.

Hypercalcemia

Abdominal pain, constipation, bone pain, decreased deep tendon reflexes, hypertension, thirst, lethargy, muscle weakness

A patient is receiving chemotherapy for the treatment of leukemia. While providing care for this patient, which clinical manifestations would indicate tumor lysis syndrome?

Altered levels of consciousness

Which should the nurse plan to monitor when providing care to a patient who is diagnosed with DiGeorge's syndrome?

Calcium

Which is the priority nursing action to decrease the risk of a transfusion reaction?

Checking the bag to ensure it is the correct blood type

The patient is receiving sodium bicarbonate intravenously (IV) for correction of acidosis secondary to diabetic coma. The nurse assesses the patient to be *lethargic, confused, and breathing rapidly*. Which is the nurse's priority response to the current situation?

Continue the infusion, because the patient is still in *acidosis*, and notify the provider.

(CC 8.4) Which change in fluid and electrolyte balance is most closely associated with aging?

Decreased olfactory function

Hyperchloremia

Deep, rapid respirations, lethargy, tachypnea, decreased cognitive ability, and hypertension

A nurse is developing a plan of care for a patient diagnosed with human immunodeficiency virus (HIV). The patient states, "I don't plan on giving up sex just because I am HIV positive." Based on this data, which is the priority nursing diagnosis for this patient?

Deficient Knowledge

A nurse is planning care for a patient with leukemia. The nurse chooses "Risk for Bleeding" as the nursing diagnosis. Which interventions support this nursing diagnosis?

Educate patient in use of soft toothbrush for oral care

The nurse is providing care to a patient diagnosed with X-linked agammaglobulinemia (XLA). Which should the nurse include in the patient's plan of care?

Immunization with inactivated polio vaccine (IPV)

Hypermagnesemia treatment

Increase fluids

The nurse is admitting a pediatric patient to the hospital with a ventroperitoneal (VP) shunt malfunction. The patient's family speaks very little English. The interpreter has arrived and the nurse is obtaining a health history from the parents and learns that the patient received the shunt at birth after a menigocele repair. Based on this data, which product should be avoided when providing care to this patient?

Latex gloves

Which is the priority nursing diagnosis for a patient diagnosed with X-linked agammaglobulinemia (XLA)?

Risk for infection

Which immune disorder should the nurse include in the plan of care for a patient who is receiving chemotherapeutic agents in the treatment of cancer?

Secondary immune deficiency

*Extra*cellular

Sodium Chloride Bicarbonate

The nurse is providing care to a patient, diagnosed with human immunodeficiency virus (HIV), with a CD4+ count of 500 cells/μL. Which classification of HIV should the nurse document for this patient?

Stage 1

Which respiratory data should the nurse anticipate when assessing a patient diagnosed with X-linked agammaglobulinemia (XLA)?

Tachypnea

Hypophosphatemia

Weakness, slurred speech, irritability, confusion, increased bleeding

Safety Alert! HYPOvolemia risk

orthostatic or postural hypotension

(Box 9.2) Causes of respiratory acidosis

• CNS depression (head trauma, oversedation, anesthesia, high cord injury) • Pneumothorax • Hypoventilation • Bronchial obstruction and atelectasis • Severe pulmonary infections • Heart failure with pulmonary edema present • Massive pulmonary embolus • Myasthenia gravis • Multiple sclerosis

(Box 9.3) Clinical Manifestations of *Respiratory Acidosis*

• Dyspnea • Restlessness • Headache • Tachycardia • Confusion • Lethargy • Dysrhythmias • Respiratory distress • Drowsiness • Decreased responsiveness

(Box 9.7) Clinical Manifestations of *Metabolic Acidosis*

• Headache • Confusion • Restlessness • Lethargy • Weakness • Stupor/Coma • Kussmaul's respiration • Nausea and vomiting • Dysrhythmias • Warm, flushed skin • Seizures • Twitching • Peripheral vasodilation

The nurse is assessing a patient who is diagnosed with human immunodeficiency virus (HIV) who presents with a rash. Which assessment question is most appropriate?

"Are you taking Bactrim?"

The nurse is caring for a patient who is receiving IV fluids post-op following *cardiac surgery*. The nurse is aware that *this patient is at risk for fluid volume excess*. The family asks why the patient is at risk for this condition. Which response by the nurse is the most appropriate?

"Fluid volume excess is common due to *increased levels of ADH in response to the stress* of surgery." Antidiuretic hormone (ADH) and aldosterone levels are commonly increased following the stress response before, during, and immediately after surgery. This increase leads to sodium and water retention. Adding more fluids intravenously can cause a fluid volume excess and stress upon the heart and circulatory system.

A pediatric patient with a history of anaphylactic hypersensitivity reactions will be discharged with a prescription for an EpiPen. Which statement is appropriate for the nurse to include in the discharge instructions for this patient and family?

"Frequently check the expiration date of the medication."

A nurse is performing an admission assessment on a patient with symptoms that indicate human immunodeficiency virus (HIV). Which question from the nurse addresses a major risk factor for contracting HIV?

"Have you ever experimented with intravenous drugs?"

The nurse is caring for a patient who is newly diagnosed with human immunodeficiency virus (HIV). The patient asks the nurse if there are ways to protect the patient's life partner from getting the HIV virus. After educating the patient, which statement indicates the need for further education?

"I know to use an oil-based lubricant to prevent spread of the disease to my partner."

A nurse has been providing a young adult patient with a history of hypersensitivity reactions. The nurse is preparing instructions on the correct methods for using an EpiPen. Which patient statement indicates understanding of the proper technique?

"I make sure the EpiPen is always available."

The nurse has completed a seminar teaching a group in the community about ways to reduce cancer risks. The nurse returns a month later to evaluate the effectiveness of the seminar. Which statements made by members of the group indicate retention and application of the material presented by the nurse to reduce the risk of developing cancer?

"I stopped using tanning booths."

The nurse completes discharge teaching for a patient with an anxiety disorder. Which patient statement indicates correct understanding of information related to respiratory alkalosis?

"I will see my counselor on a regular basis."

The nurse is providing care to a patient diagnosed with human immunodeficiency virus (HIV). Which patient statement indicates the need for further education regarding HIV management?

"I will take my medications when others can see me, even if that means taking them late."

The nurse is caring for a patient who had a bone marrow transplant for the treatment of leukemia several weeks ago. The patient requires protective isolation. Which statement by the patient's family indicates understanding of this type of isolation?

"We will encourage meticulous hand washing among all visitors."

A patient is scheduled to undergo a prostate biopsy. The patient asks the nurse what is expected immediately following the procedure. Which response by the nurse is the most appropriate?

"You will likely experience discomfort for 24-48 hours after the procedure."

Which question should the nurse to ask during a health history with an adolescent patient, accompanied by a parent, to determine immune status?

* "Are your child's immunizations up-to-date?"*

The nurse is conducting a health history for a patient who is at risk for infection. Which question is appropriate when collecting data related to the patient's past medical history?

* "Did you have your spleen removed after your car accident?"*

A patient receives the yellow fever vaccine before traveling to the Amazon Basin and asks the nurse how the vaccine provides protection. Which responses by the nurse is the most appropriate? Select all that apply.

* - "A response from yellow fever-specific T cells is activated. B cells secrete yellow fever antibodies."*(Antibodies directly attack and destroy antigens either before or after antigens invade body cells.) *- "In the lymph nodes, part of the lymphoid system, the macrophages present yellow fever antigens to T cells and B cells." * (Lymph nodes filter foreign products or antigens from lymphatic system and house and support proliferation of lymphocytes and macrophages.) *- "The initial weak infection is eliminated and the patient is left with a supply of memory T and B cells for future protection against yellow fever."* *- "Human macrophages engulf the weakened vaccine virus as if it is dangerous and antigens stimulate the immune system to attack it." *

The nurse is conducting a physical assessment for a patient with a compromised immune system. Which actions by the nurse are appropriate? Select all that apply.

* - Assessing general appearance * * - Checking joint range of motion (ROM), including that of the spine * *- Inspecting the mucous membranes of the nose and mouth for color and condition * *- Palpating the cervical lymph nodes for evidence of lymphadenopathy or tenderness *

The nurse is assessing a patient's immune system. Which findings increase the patient's risk for infection due to alterations in biochemical barriers? Select all that apply.

* - Dry mouth. * (Saliva = biochemical barrier to infection.) *- Clogged tear duct. * (Tears = biochemical.) *

The nurse is assessing a patient's immune system. Which findings increase the patient's risk for infection due to alterations in mechanical barriers? Select all that apply.

* - Dysphagia * * - urinary retention * Non-intact skin = physical

Which locations should the nurse include when discussing the storage and production of lymphocytes during an education session for novice nurses? Select all that apply.

* - Spleen * * - Thymus * * - Lymph nodes * * - Bone marrow *

Which nursing action is appropriate when assessing a patient's tonsils during a physical examination?

* Asking the patient to open the mouth and say "ah".*

A nurse is caring for a patient with who is experiencing leukocytosis. When providing care to this patient, which action by the nurse is the most appropriate?

* Assess for source of infection.* WBC = 5,000-10,000

The nurse is providing care to a patient who had the spleen removed after a car accident. Which type of infection is this patient at an increased risk for experiencing?

* Bacterial.* (The impact of a splenectomy is a loss of recognition and encapsulation of bacteria; this patient has an increased risk for a bacterial infection.)

Which type of immunoglobulin (Ig) is produced during an allergic reaction?

* IgE.* IgA = exocrine gland secretions - breast milk, tears IgD = B-cell activation IgM = initial antibody production after infection

Which nutritional deficiency often impacts a patient's ability to mount an immune response?

* Proteins.* (Nutritional status is a critical component of immune competence.) Proteins impact cellular immunity, phagocyte activity, and complements activity. Calcium = Bone health Potassium = cardiovascular

The nurse is teaching a group of patients about first-line defense against infection. Which patient statement indicates the need for further education?

*"A cut with pus is a mechanical first-line defense against infection."* (Pus or exudate indicates cellular infiltration which is a second line of defense against infection.)

The nurse is providing care to a patient who has an increased number of lymphocytes. Which explanation should the nurse provide to the patient regarding this abnormality?

*"An elevated neutrophil count indicates your body is battling a bacterial infection."* Elevated eosinophils = bacterial infection Elevated basophils = allergic reaction

The nurse is conducting a health history for a patient who is at risk for infection. Which question is appropriate when collecting data related to the patient's social history?

*"Do you smoke cigarettes?"* (social history) Are your immunizations current and up-to-date? Assesses the immunization history What type of reaction do you have with an allergy flair? Assesses the current problem Did you have your spleen removed after your car accident? Assesses the medical/surgical history

Which patient statement indicates the need for *additional education* regarding the use of sodium *bicarbonate to treat acidosis*?

*"I should use the antacid for at least 2 months."* use no longer than 2 weeks

The nurse is teaching a group of children and their parents about the prevention of *heat-related illness during exercise*. Which statement by a parent indicates an appropriate *understanding* of the preventive techniques taught during the teaching session?

*"I will have my child stop every 15-20 minutes during the activity for fluids."* During activity, stopping for fluids every 15-20 minutes is recommended.

The nurse is conducting a health history for a patient who is at risk for infection. Which question is appropriate when collecting data related to the current problem?

*"What type of reaction do you have with an allergy flair?"*

A patient is admitted to the emergency department (ED) for *dehydration*. The patient is *154 lbs*. Which *urine output* indicate the rehydration efforts for this patient have been effective?

*35 mL/hr* Expected urine output for an adult patient is 0.5 mL/kg/hr. The patient currently weighs 70 kg; therefore, adequate urine output would be at least 35 mL/hr.

Which diagnostic test should the nurse anticipate when providing care to a patient diagnosed with chronic obstructive pulmonary disease (*COPD*) to monitor acid-base balance?

*ABG*

Which physiological barriers protect the patient's body against microorganisms?

*Adequate urinary output.*

A nurse working in the emergency department (ED) is providing care for a group of patients. Which patient demonstrates a decline in immune response that typically occurs with the aging process?

*An 88-year-old with pneumonia who has a temperature of 99.5°F.*

The nurse is providing care to patient who is at an increased risk for infection due to poor dietary intake, a decreased white blood cell count, and diminished neutrophil activity. Which information in the patient's health history supports the current data?

*Anorexia nervosa.* (malnutrition causes decreased WBC and decreased neutrophil activity, which leads to increased risk for infection.)

The nurse is providing care for a patient admitted to the unit with *respiratory failure and respiratory acidosis*. Which data from the nursing history is the probable cause for the patient's current diagnoses?

*Aspiration pneumonia*

The nurse is providing care to a patient who is prescribed *furosemide* as part of the treatment for congestive heart failure (CHF). The patient's serum *potassium* level is *3.4 mEq/L*. Which food should the nurse encourage the patient to eat based on this data?

*Bananas* A potassium level of 3.4 is low, so the client should be encouraged to consume potassium-rich foods. Of the foods listed, the highest in potassium is banana.

The nurse is providing care to a patient who has a decreased neutrophil count and elevated hepatic enzymes. Which data in the patient's health history supports this laboratory data indicating an increased risk for infection?

*Cirrhosis of the liver.* ( example of hepatic disease, leads to decreased neutrophil count, which leads to increased risk for infection.)

The results of a patient's arterial blood gas sample indicate an oxygen level of 72 mmHg. Which should the nurse closely assess when providing care to this patient?

*Cognition* an oxygen level less than 74 can be due to HYPOventilation. this will change the pt's lvl of responsiveness

The nurse is caring for a patient with congestive heart failure who is admitted to the medical-surgical unit with *acute hypokalemia*. Which prescribed *medication may have contributed* to the patient's current hypokalemic state?

*Cortisol* Excess potassium loss through the kidneys is often caused by such medications as corticosteroids, potassium-wasting (loop) diuretics, amphotericin B, and large doses of some antibiotics. Cortisol is a type of corticosteroid and can cause hypokalemia.

Which clinical manifestation supports the nurse's plan of *care* focusing on *chronic respiratory acidosis*?

*Daytime sleepiness*

The nurse is analyzing the patient's ABG report, which reveals a pH of 7.15. The patient has just suffered a *cardiac arrest*. Which *consequences of this pH value* does the nurse consider for this patient?

*Decreased cardiac output* nurse knows that *severe acidosis depresses Myocardial contractility*, which leads to *decreased cardiac output*.

An older adult patient, who appears *intermittently confused*, is admitted to the hospital after a fall. Based on the current data, which is the *patient at an increased risk for developing*?

*Dehydration* During the aging process, the thirst mechanism declines. In a patient with an altered level of consciousness, this can increase the risk of dehydration and high serum osmolality.

An older adult patient, who lives in a long-term care facility, presents in the emergency department (ED) due to *fever, nausea, and vomiting over the past two days*. The patient *denies thirst*. The urine dipstick indicates a *decreased urine specific gravity*. Which medical diagnosis should the nurse anticipate when planning care for this patient?

*Dehydration* Older adult patients are less able to concentrate their urine, making them susceptible to dehydration. In addition, there is a deficit of the thirst response. However, fever, nausea, and vomiting resulting from these changes are not considered normal. The patient's symptoms of nausea and vomiting suggest decreased intake and increased output through vomiting, placing the client at risk for dehydration.

The nurse is providing care to a patient who seeks emergency treatment for *headache and nausea*. The patient works in a mill without air conditioning. The patient states, "I drink water several times each day but I seem to *sweat more than I am able to replace*." Which suggestions should the nurse provide to this patient?

*Eat something salty when drinking water*. Both salt and water are lost through sweating. When only water is replaced, the individual is at risk for salt depletion. Symptoms include fatigue, weakness, headache, and gastrointestinal symptoms such as loss of appetite and nausea. The client should be instructed to eat something salty when drinking water to help replace the loss of sodium.

The nurse is providing care to a patient whose *serum calcium levels have increased* since a surgical procedure performed three days prior. Which *intervention* should the nurse implement to *decrease the risk for the development of hypercalcemia*?

*Encourage ambulation three times a day* *Hypercalcemia can occur from immobility*. Ambulation of the client helps to prevent leaching of calcium from the bones into the serum.

Which laboratory test should the nurse anticipate for a patient who reports chronic inflammation?

*Erythrocyte Sedimentation Rate (ESR) =. Screens for the presence of the inflammatory process.* Complete blood count (CBC), with differential.* (measures total leukocytes with a breakdown of leukocyte types and amount present.) A type and cross match = for the patient who has lost blood and requires a transfusion. Varicella Titer = for the patient who is uncertain of their chicken pox status.

A patient with *acute renal failure* has *jugular vein distention*, *LE edema*, & *elevated blood pressure*. Based on this data, *which nursing diagnosis is the most appropriate*?

*Excess Fluid Volume* Jugular vein distention, edema, and elevated blood pressure are indications of excessive fluid. The diagnosis Excess Fluid Volume should be selected to guide this patient's care.

The nurse is providing care to an older adult patient who is receiving intravenous (IV) fluids at 150 mL/hr. The patient is currently exhibiting *crackles in the lungs, shortness of breath, and jugular vein distention*. Which complication of IV fluid therapy does the nurse suspect the patient is experiencing?

*Fluid volume excess* Fluid volume excess may occur when older adult patients receive intravenous fluids rapidly.

The nurse is providing care to a patient who has been vomiting for several days. The nurse knows that the patient is at *risk for metabolic alkalosis because gastric secretions have which characteristic*?

*Gastric secretions are acidic*. -metabolic alkalosis due to loss of hydrogen ions usually occurs because of vomiting or gastric suction. gastric secretions are highly acidic (ph 1-3). increased alkalinity results from the loss of acid from selective retention of bicarbonate by the kidneys as chloride is depleted

A patient is admitted to the emergency department (ED) for *fluid volume deficit*. Which *body system should the nurse focus* to determine the cause of this imbalance when assessing this patient?

*Gastrointestinal* The most common cause of fluid volume deficit is excessive loss of gastrointestinal fluids, which can result from vomiting, diarrhea, suctioning, intestinal fistulas, or intestinal drainage. Other causes of fluid losses include chronic abuse of laxatives and/or enemas.

Which scenario should the nurse provide as one in which active immunity is acquired when educating a group within the community?

*Having measles as a child.*

The nurse is reviewing lab values for a female patient suspected of having a *fluid imbalance*. Which laboratory value evaluated by the nurse supports the *diagnosis of dehydration*?

*Hematocrit 53%* The hematocrit measures the volume of whole blood that is composed of RBCs. Because the hematocrit is a measure of the volume of cells in relation to plasma, it is affected by changes in plasma volume. The hematocrit increases with severe dehydration.

(CC 8.6) The nurse prioritizes the patient with which electrolyte abnormality at greatest risk for *ventricular fibrillation*?

*Hypokalemia*

The nurse is providing care to an adult patient admitted with *dehydration* and *hyponatremia*. *Which* medical condition *supports the current nursing diagnosis of Electrolyte Imbalance*?

*Hypotonic dehydration* Hypotonic dehydration occurs when fluid loss is characterized by a proportionately greater loss of sodium than water, causing serum sodium to fall below normal levels

HYPOvolemia complications

*Hypovolemic shock* can develop as evidenced by hypotension, tachycardia, and signs of organ hypoperfusion such as cool, clammy skin, oliguria progressing to anuria (lack of urine output), decreased level of consciousness, and tachypnea.

The nurse is teaching a new mother the immune benefits of breastfeeding her newborn. Which immunoglobulin (Ig) should the nurse include as one that is passed from mother to newborn by breast milk?

*IgA.* ( breast milk and tears) IgG = placenta

The nurse is planning care for an older adult patient with *respiratory acidosis*. Which intervention should the nurse include in this patient's plan of care?

*Maintain adequate hydration*. in respiratory acidosis= drop in blood pH, reduced lvl of oxygen, & retaining of CO2. the body needs to be well-hydrated

The nurse is providing care for an adult patient who is admitted to the emergency department (ED) after passing out. The patient has been *fasting and currently has ketones in the urine*. Which acid-based imbalance should the nurse monitor the patient for based on the current data?

*Metabolic acidosis* -pt who are fasting = risk for metabolic acidosis. The body recognizes fasting as starvation & begins to metabolize its own proteins into ketones, which are metabolic acid

The nurse is providing care to a patient who is diagnosed with multisystem *fluid volume deficit*. The patient is currently experiencing *tachycardia* and *decreased urine output* along with *skin that is pale and cool to the touch*. The patient has a *decreased urine output*. Which probable cause to the patient's symptoms should the nurse include when educating the family?

*Natural compensatory mechanisms* The internal vasoconstrictive compensatory reactions within the body are responsible for the symptoms exhibited. The body naturally attempts to conserve fluid internally specifically for the brain and heart. ❄️

A patient is admitted to the emergency department for the treatment of a *drug overdose causing acute respiratory acidosis*. Which substance noted on the toxicology report is the most likely cause for the current diagnosis?

*Oxycodone* -excessive use or overdose of narcotic substances can lead to respiratory depression and respiratory alkalosis

The nurse is planning care for the patient with *acute renal failure*. The nurse plans the patient's care based on the nursing diagnosis of Excess Fluid Volume. Which assessment data supports this nursing diagnosis?

*Pitting edema in the lower extremities* The patient in acute renal failure will likely be edematous, as the kidneys are not producing urine.

A patient with metabolic acidosis has been admitted to the unit from the emergency department (ED). The patient is experiencing *confusion and weakness*. Which independent nursing intervention is the *priority*?

*Protecting the patient from injury* the pt w/ metabolic acidosis may have symptoms of drowsiness, lethargy, confusion and weakness

The nurse is providing care to a patient who is admitted after a *morphine overdose*. Which acid-base imbalance should the nurse plan this patient's care to reflect?

*Respiratory acidosis* -Morphine is a narcotic and generally acts tso decrease or suppress respirations; therefore, this patient is probably hypoventilating. The expected acid-base imbalance would be respiratory alkalosis

The client is admitted to the emergency department (ED) with symptoms of a *panic attack*. Based on this data, the nurse plans care for which health problem?

*Respiratory alkalosis*

The nurse is analyzing the I&O record for a patient being treated for *dehydration*. The patient weighs *176 lbs. and had a 24-hour intake of 2,000 mL and urine output of 1,200 mL*. Based on this data, which conclusion by the nurse is the most appropriate?

*Treatment is effective and should continue*. Urinary output is normally equivalent to the amount of fluids ingested; the usual range is 1,500-2,000 mL in 24 hours, or *40-80 mL in 1 hour* (0.5 mL/kg per hour). Patients whose intake substantially exceeds output are at risk for fluid volume excess; however, the patient is dehydrated. The extra fluid intake is being used to improve body fluid balance. The patient's output is 40 mL/hr, which is within the normal range.

Which data collected by the nurse during the assessment process *places the older adult patient at risk for dehydration*?

*Water intake of 2 glasses per day* A poor intake of water could indicate a loss of the thirst response, which occurs as a normal age-related change. Since the patient only ingests two glasses of water each day, this could indicate a reduction in the normal thirst response.

The nurse is preparing to perform a health assessment on an adult patient who has a family history of cancer. Which questions should the nurse ask the patient to assess for the early warning signs of cancer? Select all that apply

-"Have you noticed any cuts that have not healed?" -"Have you noticed any cuts that have not healed?" -"Have you experienced any problems swallowing?" -"Do you have a cough that is not associated with seasonal allergies?'

The nurse instructs a group of community members on the difference between benign and malignant neoplasms. Which participant statements indicate that teaching has been effective? Select all that apply.

-"Malignant tumors can grow back." -"Benign tumors stay in one area." -"Benign tumors grow slowly." -"Malignant tumors are easy to remove."

Hyperchloremia treatment

-0.45% NS

Hypernatremia treatment

-Fluid replacement 1/2 NS or D5W

Hyponatremia treatment

-Fluid restriction -In severe neurological symptoms, 3% NS -Oral sodium supplements -Loop diuretics

Hypocalcemia treatment

-IV calcium -Oral supplementation (calcium tablets, Tums)

Hypercalcemia treatment

-IV fluids -Loop diuretics -Bisphosphates

Hypomagnesemia treatment

-IV replacement Magnesium 2 g in 100 mL *NS* or *D5W* over 1 hr -Mg tablets -*IVP during Code Blue* situation

Hypokalemia treatment

-Increase dietary intake or supplementation -Slow-K, KCl liquid, powder or tablets -IV replacements (KCL runs, KCL aliquots): Peripheral IV: KCL 10 mEq in 100 mL NS or D5W over 1 hour Central line (PICC, triple-lumen catheter, or portacath): KCL 10 or 20 mEq in 100 mL NS or D5W to infuse over 1 hour *Never administer as IVP*

A nurse is caring for a patient who is diagnosed with skin cancer. Which nursing interventions will reduce the growth of cancer cells and support normal cell function? Select all that apply.

-Increasing calorie intake -Encouraging increased rest and sleep -Assessing normal functioning of organ systems

Hyperkalemia treatment

-Kayexalate, 50% dextrose & IV regular insulin IVP bolus -Sodium bicarbonate 50 mEq IVP -Calcium gluconate -Albuterol nebulization for 1 hour -Loop diuretics

The nurse is caring for a thin, older adult patient who is diagnosed with cancer and is receiving aggressive chemotherapy. The patient is experiencing severe side effects from the therapy and has lost 10 pounds in the past week. What should the nurse teach the patient to do? Select all that apply

-Keep a food diary and record intake. -Eat small frequent meals high in calories. -Drink liquid supplements to increase intake of nutrients -Eat cold foods rather than hot foods, because they are better tolerated.

The nurse is caring for a patient who is diagnosed with cancer. Which diagnostic tests may be helpful to assist with treatment options? Select all that apply.

-MRI -Urinalysis -Tumor markers

Hyperphosphatemia treatment

-PhosLo capsules -Oral phosphate-binding drugs -If normal renal function, IV NS and loop diuretics

The nurse is reviewing the laboratory values of a patient who is newly diagnosed with acquired immunodeficiency syndrome (AIDS). Which values should be reported to the patient's health-care provider? Select all that apply

-T4 cell count 150 -CD4 lymphocytes 12% -Viral load 11,500 copies/mL 5) WBC 6,500

The nurse is planning care for a pediatric patient diagnosed with human immunodeficiency virus (HIV). The nurse selects Risk for Infection as a priority nursing diagnosis for this pediatric patient. Based on this nursing diagnosis, which actions by the nurse are appropriate? Select all that apply

-Teaching proper food-handling techniques to the family -Assessing the health status of all visitors -Monitoring hand-washing techniques used by the family

Magnesium Foods

-halibut -almonds, cashews, NUTS -spinach -cereal, bran, shredded wheat, oatmeal -potato -PB, -BEANS, rice, lentils -avocado -yogurt -chocolate milk, milk -banana -bread -raisins -chocolate pudding

Calcium foods

-milk -orange/grapfruit -canned salmon -american cheese -broccoli -tofu -kale -ice cream -egg

Potassium foods

-spinach -baked potato -tomato juice -mushrooms -sweet potato -vegetable juice cocktail -banana -cantaloupe -low-fat milk -kidney beans -avocado -tomato -kiwi -strawberries -orange

The nurse is caring for the patient experiencing hypovolemic shock and metabolic acidosis. Which nursing actions are appropriate for this patient? (SATA)

1) Administer sodium bicarbonate. 2) Monitor ECG for conduction problems. -these are appropriate for a pt with shock 3)Keep the bed in the locked and low position. -pt recovering from shock is at risk for injury, so the bed should be kept in the locked and low position.

The nurse is providing care to a patient who is admitted with manifestations of metabolic alkalosis. Which diagnostic test findings support the admitting diagnosis? (SATA)

1) Blood pH 7.47 and bicarbonate 34 mEq/L 2)Electrocardiogram changes consistent with hypokalemia

A patient recently diagnosed with diabetes mellitus (DM) is hospitalized in diabetic ketoacidosis (DKA) after a religious fast. The patient tells the nurse, "I have fasted during this season every year since I became an adult. I am not going to stop now." The nurse is not knowledgeable about this particular religion. Which nursing actions would be appropriate? (SATA)

1) Request a consult from a diabetes educator. 2) Assess the meaning and context of fasting for this religion. 3) Encourage the patient to seek medical care if signs of ketoacidosis occur in the future.

Which nursing actions are appropriate when conducting an Allen test? (SATA)

1) Rest the patient's arm on the mattress. 2) Support the patient's wrist with a rolled towel. 3) Press the patient's *radial* and *ulnar* arteries using the index and middle fingers.

Which risk factors exhibited by the patient presenting in the emergency department (ED) would place the patient at risk for metabolic acidosis? (SATA)

1)Abdominal fistulas 2)Acute renal failure 3)Hypovolemic shock -metabolic acidosis i s a primary disorder. It usually develops during the course of another disease presence of abdominal fistulas; which cause bicarbonate loss, acute renal failure, and hypovolemic shock

The nurse notes that a pt has a positive Babinski response. For what should the nurse asses this pt? Select all that apply. 1. Alcohol abuse 2. Substance abuse 3. Multiple sclerosis 4. Parkinson's disease 5. Traumatic brain injury

1, 2, 3, 5

The nurse is preparing a teaching tool about the nervous system. Which type of tissue should the nurse identify as being a part of gray matter? Select all that apply. 1. Dendrites 2. Cells bodies 3. Myelin sheath 4. Axon terminals 5. Nodes of Ranvier

1, 2, 4

A pt is recovering from a cerebral angiogram. What care should the nurse provide to this pt? Select all that apply. 1. Monitor intravenous fluid infusion 2. Elevate the puncture site limb on a pillow 3. Maintain pressure on arterial puncture site 4. Enforce bedrest for 6-12 hours after the procedure 5. Monitor vital signs every 15 minutes for the first hour

1, 3, 4, 5

The nurse is proving care to a trauma pt who will require the rapid administration of large volumes of fluid in addition to a blood transfusion. Which gauge should the nurse use when initiating IV access for this pt? 1. 18 2. 20 3. 22 4. 24

1. 18

The nurse is planning care for a pt w/ Guillain-Barre syndrome. Which intervention will help w/ neuropathic pain? 1. Administer gabapentin 2. Turn and reposition q 2 hrs 3. Apply sequential compression devices 4. Perform passive ROM several times a day

1. Administer gabapentin

A pt is unable to feel light touch down the anterior left leg and top of the left foot. How should the nurse document this finding? 1. Anesthesia 2. Paresthesia 3. Hypoesthesia 4. Hyperesthesia

1. Anesthesia

The nurse is reviewing medical orders written for a pt w/ encephalitis. Which medication should the nurse expect to be prescribed as a priority for this pt? 1. Antiviral 2. Antibiotic 3. Antiemetic 4. Antiseizure

1. Antiviral

Which technique should the nurse use to assess a pt's CN IX Glossopharyngeal? 1. Apply a tongue depressor to the back of the throat 2.Ask the pt to read from a book or a newspaper 3. Ask the pt to smile, frown, puff out cheeks, & raise eyebrows 4. Ask the pt to follow the examiner's finger as it is moved toward the pt's nose

1. Apply a tongue depressor to the back of the throat

A pt is experiencing bilateral symmetrical muscle weakness and sensory changes of both feet and legs. What should yhe nurse expect to assess that determines the presence of Guillain-Barre syndrome? 1. Areflexia 2. Hyporeflexia 3. Hyperreflexia 4. Hyperanalgesia

1. Areflexia

The nurse is caring for a pt w/ a central venous catheter used for intermittent med administration. When flushing the catheter prior to administering the next dose of medication, which initial action by the nurse is the most appropriate? 1. Aspirating the pt's cath for blood 2. Positioning the pt in reverse Trendelenburg position 3. Flushing the cath, using as much force as required in order to clear the line 4. Obtaining a 3 mL syringe and filling it with NS for flushing the line

1. Aspirating the pt's cath for blood

The nurse suspects that a pt w/ a brain tumor is developing increased intracranial pressure. What assessment findings caused the nurse to make this conclusion? Select all that apply. 1. Ataxia 2. Nausea 3. Diarrhea 4. Vomiting 5. Headache

1. Ataxia 2. Nausea 3. Diarrhea 5. Headache

A pt is scheduled for a positron emission tomography (PET) scan. For which health problem should the nurse anticipate planning care for this pt? 1. Brain tumor 2. Cerebral bleed 3. Cranial fracture 4. Cerebral blood clot

1. Brain tumor

The nurse is providing care to a pt who is receiving total parental nutrition (TPN). During the shift assessment, the nurse notes that the pt is lethargic and has an elevated temp and WBC count. The nurse suspects the pt is septic. Which actions by the nurse are appropriate on the situation? Select all that apply. 1. Changing the IV tubing 2. Saving the remaining TPN 3. Notifying the HCP 4. Recording the lot number of the TPN 5. Replacing the TPN w/ a NS solution

1. Changing the IV tubing 2. Saving the remaining TPN 3. Notifying the HCP 4. Recording the lot number of the TPN

A pt w/ progressively deteriorating lower extremity motor and sensory function is having a lumbar puncture. What finding suggests that this pt has Guillain-Barre syndrome? 1. Elevated protein level 2. Elevated glucose level 3. Reduction in WBC count 4. Increased number of RBCs

1. Elevated protein level

The nurse notes that an older pt sways when moving from a sitting to a standing position. What is this pt at risk for experiencing? 1. Falls 2. Hypothermia 3. Altered pain sensation 4. Reduced oxygen to the brain

1. Falls

The nurse is preparing medications for a client w/ encephalitis. Which medication should the nurse question before administering? 1. Furosemide (Lasix) 2. Phenytoin (Dilantin) 3. Docusate sodium (Colace) 4. Prochlorperazine (Compazine)

1. Furosemide (Lasix)

The nurse notifies the health-care provider while caring for a pt recovering from a craniotomy as treatment for a brain tumor. What did the nurse assess to cause this alarm? Select all that apply. 1. Heart rate 52 bpm 2. Temp 99.2 F 3. Resp rate 10 and irregular 4. Urine output 200 mL over 4 hours 5. Systolic BP 198 mm Hg

1. Heart rate 52 bpm 3. Resp rate 10 and irregular 5. Systolic BP 198 mm Hg

A pt w/ a cerebral vasospasm is receiving triple H therapy. What parameter should the nurse use to determine adequacy of hemodilution? 1. Hemoglobin level=30g/dL 2. BP 154/80 mm Hg 3. Serum sodium level less than 160 mg/dL 4. Serum potassium level between 4.0-4.5 mEq/L

1. Hemoglobin level=30g/dL

The nurse is concerned that a pt's Alzheimer's disease is progressing. What finding did the nurse use to confirm this suspicion? 1. Inability to recall the word for "car" 2. Misplacement of health insurance cards 3. Unable to find sweater in the waiting room 4. Leaving the practitioner's office w/out taking prescriptions

1. Inability to recall the word for "car"

The nurse is reading the results of a single-fiber electromyography completed on a pt suspected of having myasthenia gravis. Which information would validate this pt's diagnosis? 1. Increased jitter 2. Nerve compression 3. Increased antibodies 4. Decreased muscle response

1. Increased jitter

(CC 8.3) The nurse assesses for which *clinical manifestations* in the patient with *dehydration*? (SATA)

1. Increased urine specific gravity -seen in pt w/ decreased renal perfusion, dehydration or in response to ADH secretion 2. Increased serum BUN 3. Increased serum osmolality -seen in hyperglycemia, hypernatremia, & severe dehydration

The nurse is reviewing discharge instructions for a pt w/ myasthenia gravis. What should the nurse emphasize regarding medications? 1. Keep extra doses of medication in the car. 2. Store extra doses of medication in the refrigerator. 3. Take an extra dose od medication before leaving the house 4. Pack prescribed medications in a suitcase before flying on an airplane

1. Keep extra doses of medication in the car.

The nurse is admitting a pt experiencing photophobia and nuchal rigidity. For which diagnostic test should the nurse prepare this pt? 1. Lumbar puncture 2. Evoked potentials 3. CT scan w. contrast 4. Electroencephalogram

1. Lumbar puncture

The nurse notes that a pt has been diagnosed w/ trigeminal neuralgia. What should the nurse expect the pt to be experiencing? 1. Pain 2. Nausea 3. Sensory deficit 4. Motor weakness

1. Pain

The nurse is assessing an IV insertion site noting redness, warmth, and mild swelling. The pt reports a burning pain along the course of the vein during medication administration. Which term should the nurse use when documenting these findings in the medical record? 1. Phlebitis 2. Infiltration 3. Extravasation 4. Inflammation

1. Phlebitis

During an assessment the nurse suspects that pt should be evaluated for myasthenia gravis. What did the nurse assess to make this clinical determination? Select all that apply. 1. Ptosis 2. Diplopia 3. Abdominal pain 4. Left leg weakness 5. Epigastric burning

1. Ptosis 2. Diplopia

The nurse is preparing to assess an older pt's neurologic status. What should the nurse keep in mind during this assessment? 1. Reaction time is slower 2. Flexibility is maintained 3. Pain sensation is heightened 4. Body movements are quicker

1. Reaction time is slower

A pt returns to the community clinic after being diagnosed w/ Parkinson's disease. What should the nurse expect to see documented in the pt's medical record to support this diagnosis? 1. Rigidity w/ ambulation 2. Unremarkable electroencephalogram 3. Results of serum potassium and calcium levels 4. Integrity of cerebral vessels after a cerebral angiogram

1. Rigidity w/ ambulation

The nurse notes that a pt has ataxia. Which test should the nurse use to gain more information about this pt's gait? 1. Romberg 2. Patellar reflex 3. Plantar flexion 4. Achilles reflex

1. Romberg

A pt w/ Guillain-Barre syndrome is receiving plasmapheresis. What finding should the nurse identify as being a complication of this treatment? 1. Septicemia 2. Flu-like symptoms 3. Aseptic menigitis 4. Acute renal failure

1. Septicemia

A pt experiences a cramp in the right thigh. Which spinal tract is responsible for communicating to the thigh muscle to contract? 1. Somatic motor division 2. Visceral motor division 3. Somatic sensory division 4. Visceral sensory division

1. Somatic motor division

The nurse is providing care to a patient who is receiving a blood transfusion. Ten min after the infusion is initiated, the pt reports a headache. Upon further assessment the nurse notes that the pt is experiencing dyspnea and feels warm to the touch. Which is the prioity nursing action by the nurse? 1. Stop the transfusion. 2. Prepare for a full resuscitation. 3. Notify the HCP. 4. Decrease the rate of the transfusion.

1. Stop the transfusion.

The nurse notes that a pt w/ myasthenia gravis is experiencing bulbar manifestations, On what should the nurse focus when assessing this pt? Select all that apply. 1. Swallowing 2. Eye opening 3. BP 4. Tongue pressure 5. Head and neck movement

1. Swallowing 3. BP 4. Tongue pressure 5. Head and neck movement

The nurse is planning discharge instructions for a pt recovering from surgery to debulk a brain tumor. Why should self-monitoring of capillary blood glucose level be a part of this teaching? 1. Taking steroid medication 2. Manipulation of cerebral tissue 3. Interruption of glucose regulation in the brain 4. Alteration in oral intake from chemotherapy

1. Taking steroid medication

Which pts may benefit from central IV access? Select all that apply. 1. The pt receiving caustic IV therapy 2. The pt requiring long-term IV therapy 3. The pt who is afraid of needles and does not want a catheter in the peripheral extremity 4. The pt requiring numerous IV infusions that are not compatible and cannot be infused together 5. The unstable pt requiring reliable IV access for administration of meds required immediately.

1. The pt receiving caustic IV therapy 2. The pt requiring long-term IV therapy - 4. The pt requiring numerous IV infusions that are not compatible and cannot be infused together 5. The unstable pt requiring reliable IV access for administration of meds required immediately.

A pt w/ Parkinson's disease is prescribed carbidopa/levodopa (Sinemet). Which clinical manifestation should the nurse expect to be most affected w/ this medication? 1. Tremors 2. Mood instability 3. Impaired balancec 4. Behavioral changes

1. Tremors

The nurse is caring for a pt w/ a central venous catheter. Which nursing actions should the nurse implement to prevent an air embolism? Select all that apply. 1. Using Luer-locked connections 2.Frequently checking connections 3. Wearing sterile gloves when accessing any connections 4. Clamping catheters and injection sites when not in use 5. Placing the pt in low-Fowler position to remove the CVC

1. Using Luer-locked connections 2.Frequently checking connections - 4. Clamping catheters and injection sites when not in use

The nurse is reviewing orders written for a pt w/ myasthenia gravis. Which medication order should the nurse question before administering? Select all that apply. 1. Verapamil 2. Furosemide 3. Erythromycin 4. Nicotine patch 5. Warfarin sodium

1. Verapamil 3. Erythromycin 4. Nicotine patch

The nurse is performing venipuncture to initiate IV therapy. Which indicators should the nurse use when choosing the site for IV therapy? Select all that apply. 1. When choosing a straight vein 2. Avoiding a sclerotic vein 3. Looking for sites distal to joints 4. Using the dominant arm, whenever possible 5. Choosing a vein that is visible in addition to palpable

1. When choosing a straight vein 2. Avoiding a sclerotic vein 3. Looking for sites distal to joints

The nurse suspects that a pt is in the premonitory phase of a migraine headache. What findings did the nurse use to make this clinical decision? Select all that apply. 1. Yawning 2. Confusion 3. Food cravings 4. Flashing lights 5. Increased urine output

1. Yawning 3. Food cravings 5. Increased urine output

Colloid

1.) *Albumin*: 5% or 25% 2.) *Dextran* (polysacch.): 40 kDa or 70 kDa 3.) *HES* (synthetic starch): 6% or 10% 4.) *Mannitol* (alcohol sugar): 5% or 25%

The community nurse visits the home of a young child who is home from school because of sudden onset of *nausea, vomiting, and lethargy*. The nurse suspects *acute renal failure*. Which *clinical manifestations* support the nurse's suspicions?

1.) *E*dema 2.) *H*ematuria 3.) *E*levated blood pressure -Pediatric manifestations of *acute renal failure* characteristically begin w/ a healthy child who suddenly becomes ill with nonspecific symptoms that indicate a significant illness or injury. These symptoms may include any combination of the following: *nausea, vomiting, lethargy, edema, gross hematuria, oliguria, & HTN

The school nurse is preparing a class session for high school students on ways to *maintain fluid balance* during the summer months. Which interventions should the nurse recommend (SATA)

1.) *Reduce* the intake of *coffee & tea*. 2.) Drink *more fluids during hot weather*. 3.) Drink *flat cola or ginger ale if vomiting*. -Actions to *prevent fluid volume deficit* during the summer months *include increasing fluid intake, drinking flat cola or ginger ale if vomiting*, and *reducing* the intake of *coffee & tea*.

The nurse is concerned that an *older adult* patient is at risk for developing *acute renal failure*. Which information in the patient's history support the nurse's concern?

1.) Diagnosed with *hypotension* 2.) *Recent aortic valve replacement surgery* 3.) Prescribed *high doses of intravenous antibiotics* -Older adults develop acute renal failure more frequently because of the higher incidence of serious illnesses, HYPO-TN, major surgeries, diagnostic procedures, and treatment with nephrotoxic drugs. Decreased kidney function associated with aging also puts the older patient at risk for kidney failure. HYPO-TN, scheduled for aortic valve replacement surgery, and receiving high doses of intravenous antibiotics increase this patient's risk for developing acute renal failure.

The nurse is providing care to a patient who is exhibiting *clinical manifestations of a fluid and electrolyte deficit*. Based on this data, which health-care provider prescriptions does the nurse prepare to implement? (SATA)

1.) Initiate *hypodermoclysis* -Hypodermoclysis, fluid administered subcutaneously, may be employed as a fluid delivery method, especially among older adults. 2.) Closely monitor patient's *I&O's* -Monitoring patient's intake and output is one of several ways to assess the patient's fluid status. 3.) Initiate *IV therapy* -IV fluids may be ordered for the patient with a fluid volume deficit if replacement oral fluids can't be taken in sufficient quantity.

A pt sustains an injury to the left temporal lobe. Which body functions should the nurse expect to be affected by this injury? Select all that apply. 1. Body position 2. Long-term memory 3. Auditory perception 4. Receptive speech center 5. Expressive speech center

2, 3, 4

The nurse is conducting a medical history w/ a pt experiencing headaches. What information should be included in this history? Select all that apply. 1. Date of last lipid screen 2. Past surgical procedures 3. Recent diagnostic studies 4. Treatment for chronic illnesses 5. Date of last influenza vaccination

2, 3, 4

The nurse is completing a Mini Mental Status Examination with a pt. What should the nurse ask to evaluate remote memory? 1. "Where did you park your car?" 2. "Where did you work in the 1970s?" 3. "Remember the colors red, green, blue, and yellow." 4. "What television show was on this morning during breakfast?"

2. "Where did you work in the 1970s?"

A client is diagnosed with myasthenia gravis. What should the nurse explain about this disease process? 1. "Your nerve endings are worn out." 2. "Your body does not recognize the neurotransmitter needed for movement." 3. "Your body does not make enough of the neurotransmitter needed for movement." 4. "Your nerves have lost their protective coverings and impulses cannot reach body areas."

2. "Your body does not recognize the neurotransmitter needed for movement."

The nurse is caring for a pt receiving IV meds. After infusing an IV antibiotic, the nurse assesses the IV site and finds it to be red and edematous, and the pt is reporting pain at the site. Which would the nurse document in the nursing notes regarding the infiltration? Select all that apply. 1. Incident report 2. Actions taken to correct the problem 3. Size and location of erythematous area 4. HCP notification and any orders received 5. Amount of fluid infused per shift on the I&O record

2. Actions taken to correct the problem 3. Size and location of erythematous area

A pt's BP increases after hearing that diagnostic tests for a health problem have to be repeated. Which receptor of the sympathetic nervous system is responsible for this BP change? 1. Beta 2. Alpha 3. Nicotinic 4. Muscarinic

2. Alpha

A pt is diagnosed w/ a subarachnoid hemorrhage caused by a cerebral aneurysm that has a wide neck and tortuous vascular anatomy. For which procedure should the nurse prepare teaching material for this pt? 1. Aneurysm coiling 2. Aneurysm clipping 3. Reinforcing aneurysm wall 4. Evacuation of the hematoma

2. Aneurysm clipping

A pt is being prepared for a tensilon test. What should the nurse ensure is available prior to the beginning of this test? 1. Oxygen 2. Atropine sulfate 3. Intravenous fluids 4. Nasogastric suction

2. Atropine sulfate

The nurse notes that a pt's primary complaint is burning pain on the right side of the face. What should the nurse realize this pt is describing? 1. Referred pain 2. Atypical pain 3. Vascular compression 4. Peripheral neuropathy

2. Atypical pain

A pt is scheduled for an electroencephalogram. What preprocedure information should the nurse emphasize with this pt? 1. Avoid washing hair for 2 days prior to the test 2. Avoid caffeine for 8-12 hours before the test 3. Restrict the intake of fluids for 6 hours after the test 4. Apply minimal products to the hair the morning of the test

2. Avoid caffeine for 8-12 hours before the test

The nurse suspects that a pt w/ photophobia is experiencing encephalitis. What observation of the pt's behavior caused the nurse to make this clinical decision? 1. Closed the eyes 2. Covered the ears w/ both hands 3. Placed both hands in coat pockets 4. Rubbed the thighs w/ both hands

2. Covered the ears w/ both hands

The health-care professional is identifying the best course of treatment for a pt a/ a brain tumor. What should be considered when selecting chemotherapeutic agents for this pt? 1. Cause mild neutropenia 2. Cross the blood-brain barrier 3. Restrict weight loss and hair loss 4. Limit the development of mucositis

2. Cross the blood-brain barrier

During a home visit the nurse considers physical therapy for a pt recovering from encephalitis. What would be the best explanation for this referral? 1. Rehabilitation from hemiparesis 2. Deconditioning from extended bedrest 3. Improve use of limbs bc of paresthesias 4. Improve balance bc of cerebellum dysfunction

2. Deconditioning from extended bedrest

A pt w/ Guillain-Barre syndrome loses respiratory function 3 weeks into the acute phase. When should the nurse expect respiratory function to return in this pt? 1. During the plateau phase 2. Early in the recovery stage 3. At the end of the plateau phase 4. At the end of the recovery stage

2. Early in the recovery stage

A pt has been experiencing numbness of the right hand. Which diagnostic test should the nurse anticipate being prescribed for this pt? 1. Myelogram 2. Evoked potentials 3. Electroencephalography 4. Magnetic resonance imaging

2. Evoked potentials

The nurse is conducting a physical examination on a female pt experiencing trigeminal neuralgia. What observation indicates that the pain is triggered by hygienic practices? 1. Limited talking 2. Hair not combed 3. Wearing tennis shoes 4. Not wearing makeup

2. Hair not combed

The nurse interrupts unlicensed assistive personel who is assisting a pt w/ Parkinson's disease w/ breakfast. Which observation caused the nurse to immediately intervene? 1. Pt sitting out of bed in a chair 2. Head of bed raised to 30 degrees 3. Thickener added to liquid menu items 4. Oral suction catheter equipment turned on

2. Head of bed raised to 30 degrees

The staff development trainer is preparing orientation materials for new staff hired to care for pts w/ seizure disorders. Which type of seizure can occur in any age group? Select all that apply. 1. Absence 2. Myoclonic 3. Tonic-clonic 4. Simple partial 5. Complex partial

2. Myoclonic 3. Tonic-clonic 4. Simple partial

A pt is scheduled for surgery to treat trigeminal neuralgia. Which medication should the nurse expect to be prescribed for this pt? 1. Plasmapheresis 2. Percutaneous rhizotomy 3. Stereotactic radiosurgery 4. Microvascular decompression

2. Percutaneous rhizotomy

A pt comes to the community clinic complaining of having a fever. What findings should suggest to the nurse that the pt is experiencing meningitis? Select all that apply. 1. Eye tearing 2. Photophobia 3. Opisthtonos 4. Nuchal rigidity 5. Auditory hallucinations

2. Photophobia 3. Opisthtonos 4. Nuchal rigidity

A pt w/ myasthenia gravis has lost 6 kg of weight over the last 2 months. What should the nurse suggest to improve this pt's nutritional status? 1. Eat 3 large meals per day 2. Plan medication doses to occur before meals 3. Restrict drinking fluids prior to and during meals 4. Increase the amount of fat and carbs in meals

2. Plan medication doses to occur before meals

During morning care a pt w/ a seizure disorder asks why the room has suddenly turned green. What should the nurse do? 1. Ask the pt to explain 2. Prepare for a seizure to begin 3.Turn on the overhead room lights 4. Document visual hallucinations present

2. Prepare for a seizure to begin

A pt has been experiencing a tonic-clonic seizure for 5 minutes. What should the nurse do first? 1. Assess carotid pulse 2. Prepare to insert an airway 3. Provide rescue breathing 4. Insert an intravenous access line

2. Prepare to insert an airway

The nurse adds a med to an IV fluid container to be hung on the pt's existing IV line. Which is the 1st action the nurse takes after adding the med to the container? 1. Connect the bag to the tubing. 2. Rotate the bag to distribute the medication. 3. Place a completed medication-added label to the bag. 4. Connect the bag to new tubing and flush the air from the tubing.

2. Rotate the bag to distribute the medication.

The nurse needs to provide a strong stimulus to illicit a response; however, the pt drifts back to unresponsiveness. What term should the nurse use to document this pt's level of responsiveness? 1. Coma 2. Stupor 3. Lethargic 4. Conscious

2. Stupor

A pt is scheduled for a myelogram prior to having spinal fusion surgery. What should the nurse instruct the pt to do prior to this test? 1. Shower w/ antiseptic soap 2. Take nothing by mouth for 4 hours before the test 3. Take an over-the-counter analgesic before arriving for the test 4. Restrict the intake of caffeine products for 24 hours before the test

2. Take nothing by mouth for 4 hours before the test

The nurse working in the ED is caring for a pt who experienced deep-thickness burns over 40% of the body and is in shock. Which IV prescription does the nurse anticipate for this pt? 1. Nutrient solutions 2. Volume expanders 3. Electrolyte solutions 4. Total parental nutrition

2. Volume expanders

intake

2500 mL/day metabolism= 400ml food= 500 mL drink= 1600 mL

Which IV fluid should the nurse prepare when a pt requires a hypotonic solution? 1. 0.9% NS 2. 5% dextrose in water 3. 0.33& sodium chloride 4. 5%dextrose in Lactates ringers

3. 0.33& sodium chloride

The nurse is initiating IV therapy for an adult pt who requires IV fluid infusion for 2-3 days and might require blood administration. Which would the nurse choose as the best option for IV catheterization? 1. Butterfly 2. Huber needle 3. Angiocatheter 4. Implantable venous access device

3. Angiocatheter

The nurse is setting up an IV infusion on an electronic infusion pump for a pt recently admitted to the unit. After leaving the room, the pump alarms and reads high pressure. Which is the priority action by the nurse? 1. Resetting the pump to resume infusion 2. Asking the pt if the pump has been tampered with in any ways 3. Assessing the IV site and the tubing for kinks or closed roller clamps 4. Discontinuing the pt's IV access and restarting in a different area

3. Assessing the IV site and the tubing for kinks or closed roller clamps

A pt w/ meningitis is prescribed a cooling blanket. What should the nurse explain as being the purpose of this device? 1. Relieves pain 2. Increases cerebral venous outflow 3. Decreases oxygen demand in the brain 4. Reduces the transmission of the infection

3. Decreases oxygen demand in the brain

A pt recovering from a lumbar puncture rates a headache as being 8 on a pain scale of 0-10. What should the nurse do while waiting for the health-care provider to prescribe pain medication? 1. Raise the head of the bed 2. Assist to sit out of bed in a chair 3. Encourage increasing oral fluid intake 4. Turn on the side and massage the lower spine

3. Encourage increasing oral fluid intake

Which component should the nurse anticipate will be prescribed for a pt w/ an elevated prothrombin time and international normalized ratio who is at an increased risk for bleeding? 1. Platelets 2. Albumin 3. Fresh frozen plasma 4. Packed RBCs

3. Fresh frozen plasma

The nurse is concerned that a pt is at risk for trigeminal neuralgia. What information in the pt's medical record did the nurse use to make this clinical decision? 1. Has a BMI of 34 2. Takes birth control pills 3. Hx of HTN 4. Works as a computer operator

3. Hx of HTN

A pt is diagnosed w/ a continuous headache syndrome. What treatment should the nurse expect to be prescribed for this pt? 1. Antibiotics 2. Diuretic therapy 3. Intravenous fluids 4. Sodium restriction

3. Intravenous fluids

The nurse is caring for a pt who is to have a peripherally inserted central catheter line inserted tomorrow afternoon. The pt's current peripheral access line is infiltrated, and needs to be restarted. Which site would the nurse avoid using> 1. Radial vein 2. Cephalic vein 3. Median cubital vein 4. Dorsal metacarpal veins

3. Median cubital vein

During a home visit the nurse is concerned that the pt w/ Alzheimer's disease is deteriorating. What pt observation caused the nurse to have this concern? 1. Sitting in a chair watching a television program 2. Staying away from the door leading to the back yard 3. No recognition of bowel incontinence during the visit 4. Eating cheese & crackers placed on a table near the living room chair

3. No recognition of bowel incontinence during the visit

When removing a pt's cantral line dressing, which action by the nurse is the priority? 1. Applying sterile gloves 2. Inspecting the insertion site for signs of infection 3. Pulling the tape off in the direction of the cath 4. Pressing the cath into the skin while removing the tape

3. Pulling the tape off in the direction of the cath

Which aspect of IV therapy could the nurse safelt delegate to the unlicensed assistive personnel? 1. Changing the IV site dressing on the pt's left hand 2. Watching the IV insertion site of the pt who complained of pain at the site 3. Reporting pt's complaints of pain or leakage from the IV site when bathing the pt 4. Replacing pt's IV solution when bag runs dry if it is only D5W, without medications added

3. Reporting pt's complaints of pain or leakage from the IV site when bathing the pt

The nurse suspects that a pt is experiencing Parkinson's disease. What did the nurse assess to make this clinical determination? 1. Photophobia 2. Nuchal rigidity 3. Slow movements 4. Elevated body temperature

3. Slow movements

The nurse is preparing material for a community health care fair. What should the nurse identify as being the most common cause of headache pain in adults? 1. Mixed 2. Cluster 3. Tension 4. Migraine

3. Tension

A pt w/ Guillain-Barre syndrome asks how the illness develops. What should the nurse respond about the pathophysiology of this disorder?" 1. "An infection eats away at the nerve endings." 2. "An infection enters the spinal cord and erodes the nerves at the roots." 3. "The nerves are killed by infiltration of your body's WBCs used to fight an infection." 4. "After an infection your immune system created antibodies that affect the covering of the nerves."

4. "After an infection your immune system created antibodies that affect the covering of the nerves."

Family members are concerned because a pt has been confused over the last few days. What should the nurse respond to the family? 1. "Confusion is a normal sign of aging." 2. "Older people get confused at the end of the day." 3. "Unless the pt falls, there is nothing to worry about." 4. "Confusion can mean many things that should be evaluated."

4. "Confusion can mean many things that should be evaluated."

A pt seeks treatment for progressively deteriorating motor and sensory function. What question is essential for the nurse to ask when completing this pt's health hx? 1. "Have you been around any small children?" 2. "When was the last time you had anything to eat?" 3. "When was the last time you traveled out of the country?" 4. "Have you recently experienced any lung or stomach infections?

4. "Have you recently experienced any lung or stomach infections?

A pt w/ a seizure disorder asks the purpose of staying awake all night prior to having an electroencephalogram in the morning. What should the nurse explain to this pt? 1. "You will be expected to sleep during the test." 2. "Most people w/ seizure disorders receive too much sleep." 3. "This is the only way to prove that you really have a seizure disorder." 4. "Sleep deprivation can cause a seizure, which will be helpful during the test."

4. "Sleep deprivation can cause a seizure, which will be helpful during the test."

Which IV fluid should the nurse prepare when a pt requires a hypertonic solution? 1. 0.9% NS 2. 2.5% dextrose in water 3. 0.33% sodium chloride 4. 5% dextrose in Lactated ringers

4. 5% dextrose in Lactated ringers

The nurse notes that a pt w/ Guillain-Barre syndrome sweats profusely. What should the nurse do about this finding? 1. Place on a cooling blanket 2. Notify the health-care provider 3. Monitor body temp q 2 hrs 4. Change linen and gown and keep comfortable

4. Change linen and gown and keep comfortable

A pt w/ type 2 diabetes mellitus is scheduled for a CT scan w/ contrast. What should the nurse specifically instruct the pt to prepare for this test? 1. Restrict fluids for 12 hours prior to the scan 2. Drink 2L of fluid the day before the scan 3. Limit the intake of meat & dairy products prior to the scan 4. Do not take Metformin for 48 hours prior to and after the scan

4. Do not take Metformin for 48 hours prior to and after the scan

The nurse is planning care for a pt w/ meningitis. What teaching material should be prepared to explain the prescribed treatment for this disorder? 1. Fluid restriction 2. Low-fat, low-calorie diet 3. Over-the-counter analgesics 4. Long-term antibiotic therapy

4. Long-term antibiotic therapy

Which component should the nurse anticipate will be prescribed for a pt w/ acute blood loss? 1. Platelets 2. Albumin 3. Fresh frozen plasma 4. Packed RBCs

4. Packed RBCs

The nurse is caring for a pt w/ a medical diagnosis of increased ICP. Which IV fluid order would the nurse accept w/out questioning? 1. Run NS at 125 mL/hr 2. Run half-NS at 200 mL/hr 3. Run 5% dextrose in water at 80 mL/hr 4. Run 5% dextrose in 0.45% NaCl at 75 mL/hr

4. Run 5% dextrose in 0.45% NaCl at 75 mL/hr

The nurse is administering a blood transfusion to an adult pt. The pt reports feeling cold and is shivering 15 min after the initiation of the transfusion. The pt's BP had decreased since the last assessment. Which is the nurse's priority action? 1. Notify the HCP 2. Monitor the BP q 5 min 3. Stop the blood infusion, and run the NS on the other side of the Y tubing. 4. Stop the blood infusion, and remove the tubing from the IV catheter, replacing it with NS

4. Stop the blood infusion, and remove the tubing from the IV catheter, replacing it with NS

A home health nurse is conducting home visits for several patients who are diagnosed with acquired immunodeficiency syndrome (AIDS). Which patient would the nurse see first?

A patient with PCP who called the office this morning to report a new onset of fever, cough, and shortness of breath

17. The family of a patient with a traumatic brain injury asks why the bed side rails are padded. What should the nurse explain to the family? 1) "There is a risk for seizure activity after a head injury." 2) "The padding prevents injury when turning the patient." 3) "The padding prevents the patient from climbing out of bed." 4) "The padding ensures the side rails are kept elevated at all times."

ANS: 1 Feedback 1 After a head injury, the patient should be on seizure precautions because of the risk of seizure activity. 2 Padding is not placed on a bed to prevent injury when turning the patient. 3 Padding is not used to prevent the patient from climbing out of the bed. 4 Padding is not used to ensure that the side rails remain elevated at all times. This would be a form of physical restraint necessitating a health-care provider's order.

A victim of a motor vehicle crash is diagnosed with a neck whiplash. Which manifestation should the nurse expect to assess in this patient? 1) Bladder dysfunction 2) Loss of proprioception 3) Ipsilateral loss of motor function 4) Pain below the level of the injury

ANS: 1 1 A hyperextension injury causes central cord syndrome. A clinical manifestation of this syndrome is bladder dysfunction. 2 Posterior cord syndrome is caused by acute compression. A manifestation of this syndrome is a loss of proprioception. 3 Brown-Sequard syndrome is caused by a penetrating injury. A manifestation of this syndrome is an ipsilateral loss of motor function. 4 Anterior cord syndrome is caused by acute compression from bony fragments or acute disk herniation. A manifestation of this syndrome is pain below the level of the injury.

The nurse directs a patient with a herniated disk to notify the health-care provider immediately if the patient experiences which presentation? 1) New onset urinary incontinence 2) Sleepiness after taking pain medication 3) Difficulty with having a bowel movement 4) Transient pain with walking and changing position

ANS: 1 1 Common complications of a herniated disk include loss of bladder control. The development of this complication may require surgical intervention to prevent permanent loss of function. 2 Depending upon the pain medication, sleepiness may be expected. 3 Difficulty with having a bowel movement indicates constipation, which can be addressed with increased roughage and fluids. 4 Transient pain with walking and changing position is a manifestation of a herniated disk.

The nurse notes crusting of secretions around the pins of a patient with a Halo device. What action should be taken at this time? 1) Wrap the pins with gauze soaked with normal saline 2) Gently pick the crusting off of the pins with a dry gauze pad 3) Apply gauze soaked with hydrogen peroxide around the pins 4) Syringe 1/2 strength hydrogen peroxide and sterile water to the crusted areas

ANS: 1 1 If crusting is noted, wrap gauze soaked with normal saline around the pin site for 15 minutes. After removing the gauze, use a clean cotton-tipped applicator to gently remove the crust from the pin site. 2 The crusting should not be picked off the pins. 3 Hydrogen peroxide is not used to remove crusting from the pins. 4 Solutions should not be syringed on the pins. Hydrogen peroxide is not used to remove crusting from the pins.

A patient is suspected as having a spinal cord tumor. Which diagnostic test should the nurse expect to be prescribed to definitively diagnose this patient's problem? 1) MRI 2) Myelogram 3) Cerebral angiogram 4) Nerve conduction studies

ANS: 1 1 MRI is considered to be the gold standard for examining spinal structures. 2 A myelogram may be used if an MRI is not available; however, the myelogram is not identified as being the gold standard for the examination of spinal structures. 3 A cerebral angiogram would not be indicated for a spinal cord tumor. 4 Nerve conduction studies would not be indicated for a spinal cord tumor. However, if a tumor is ruled out, nerve conduction studies might be indicated to determine nerve damage and function.

A patient with low back pain asks why nerve conduction studies are prescribed. What explanation should the nurse provide to the patient relative to this diagnostic test? 1) "It measures damage to nerves." 2) "It shows pressure on nerves from herniated disks." 3) "It measures electrical impulses within muscle tissue." 4) "It shows the structure of the vertebrae and joint outlines."

ANS: 1 1 Nerve conduction studies (NCS) measure the electrical nerve impulse that indicates damage to the nerve. 2 A myelogram shows pressure on the spinal cord or nerves from herniated disks. 3 Electromyography (EMG) measures the electrical impulse within muscle tissue. 4 X-rays show the structure of the vertebrae and joint outlines.

A patient has a 4 cm spinal cord tumor located at T10. Which therapeutic treatment should the nurse anticipate will be ordered for this patient? 1) Surgery 2) Radiation 3) Chemotherapy 4) Stereotactic radiosurgery

ANS: 1 1 Surgery is usually the first step in treating tumors that can be removed with an acceptable risk of nerve damage. 2 Radiation therapy is used following an operation to eliminate the tumor remnants or to treat inoperable tumors. 3 Chemotherapy has not been proven effective for most spinal cord tumors. 4 Stereotactic radiosurgery is currently being studied for spinal cord tumors.

5. A patient in a barbiturate coma for increased intracranial pressure (ICP) has audible gurgling through the endotracheal tube. What should the nurse do first before suctioning this patient? 1) Administer 100% oxygen 2) Elevate the head of the bed 3) Interrupt sedative administration 4) Place the head in a neutral position

ANS: 1 Feedback 1 Administering 100% oxygen just prior to suctioning is performed to prevent hypoxia, which can occur during the interruption of mechanical ventilation. 2 The patient's head of the bed should already be elevated. 3 Interrupting sedative administration would be appropriate if a complete neurologic assessment is going to be conducted. 4 The head should already be in the neutral position.

A patient with low back pain asks what nonmedical treatments can be used to help with the discomfort. Which complementary and alternative therapies should the nurse recommend to this patient? Select all that apply. 1) Yoga 2) Qi gong 3) Acupuncture 4) Massage therapy 5) Chiropractic treatments

ANS: 1, 2, 3, 4 1. Yoga can be used as part of a general health regimen, to cope with illness, to improve physiological balance, or to increase relaxation; techniques include physical postures and breathing techniques with either a focused attention or an open attitude toward distractions. 2. Qi gong is an ancient Chinese discipline involving physical and mental exercises that focus on specific parts of the body. 3. Acupuncture is the insertion of small needles or exerting pressure on "energy" points in the body; the patient is supposed to experience a feeling of fullness, numbness, tingling, and warmth. 4. Massage therapy consists of alternating levels of concentrated pressure on the areas of spasm; once pressure is applied, it should not vary for 10 to 30 seconds. Massage also leads to increased endorphin levels (chemicals associated with decreased pain and increased euphoria) that are effective in chronic pain management. 5. Chiropractic treatments are not identified as a complementary or alternative therapy for low back pain.

A patient with amyotrophic lateral sclerosis (ALS) is being prepared for discharge. What teaching would be essential for the family to receive prior to taking the patient home? Select all that apply. 1) Skin care 2) Aspiration precautions 3) Recognizing exacerbations 4) Lower extremity circulation 5) Reporting changes in continence

ANS: 1, 2, 4 1. Amyotrophic lateral sclerosis is a progressive disease that leads to the inability to move. Complications include pressure ulcers. 2. Amyotrophic lateral sclerosis is a progressive disease that leads to the inability to move. Complications include aspiration of food or fluid, respiratory failure, and pneumonia. 3. Exacerbations occur with multiple sclerosis and not ALS. 4. Amyotrophic lateral sclerosis is a progressive disease that leads to the inability to move. Complications include deep vein thrombosis (DVT) and pulmonary embolism (PE). 5. Changes in continence would be a potential complication of a herniated disk.

A patient with a herniated disk is prescribed conservative treatment. For which anticipated treatments should the nurse prepare to instruct this patient? Select all that apply. 1) Reducing body weight 2) Avoiding painful positions 3) Engaging in aerobic activities 4) Performing planned exercises 5) Taking pain medication as prescribed

ANS: 1, 2, 4, 5 1. If obesity is a contributing factor, weight-loss measures are also indicated. 2. Conservative treatment consists of avoiding painful positions. 3. Conservative treatment consists of avoiding painful positions, which might include aerobic activities. 4. Conservative treatment consists of following a regimen of planned exercise. 5. Conservative treatment consists of following a regimen of pain medications.

25. The nurse suspects a patient is experiencing an ischemic stroke of the basilar artery. What manifestations did the nurse most likely assess in this patient? Select all that apply. 1) Ataxia 2) Nausea 3) Dysphasia 4) Inability to swallow 5) Difficulty with speech

ANS: 1, 2, 4, 5 Feedback 1. Manifestations of basilar artery syndrome include ataxia. 2. Manifestations of basilar artery syndrome include nausea. 3. Dysphagia is a manifestation of left middle cerebral artery syndrome. 4. Manifestations of basilar artery syndrome include difficulty swallowing. 5. Manifestations of basilar artery syndrome include difficulty in the articulation of speech.

24. A patient is demonstrating signs of increasing intracranial pressure (ICP). What physical actions should the nurse take to reduce this pressure? Select all that apply. 1) Placing the head in a neutral position 2) Turning into a left side-lying position 3) Raising the head of the bed 60 degrees 4) Elevating the foot of the bed 45 degrees 5) Placing supine with a pillow under the head

ANS: 1, 3 Feedback 1. Positioning a patient so that the neck is in a neutral position assists in facilitating venous drainage from the head. 2. Positioning a patient so that hip flexion is minimized assists in facilitating venous drainage from the head. The side-lying position uses hip flexion to maintain the position. 3. Blood volume in the intracranial space may be decreased by raising the head of the bed to greater than 45 degrees to facilitate drainage of venous blood through the jugular venous system. 4. Elevating the foot of the bed would increase ICP. 5. The supine position with a pillow under the head would increase ICP.

The nurse notes that a patient with low back pain is experiencing radiculopathy. What should the nurse expect when assessing this patient? Select all that apply. 1) Pain 2) Edema 3) Weakness 4) Numbness 5) Inability to control motor movement

ANS: 1, 3, 4, 5 1. Radiculopathy is nerve root compression and can result in pain in the affected extremity. 2. Edema is not a manifestation of radiculopathy. 3. Radiculopathy is nerve root compression and can result in weakness in the affected extremity. 4. Radiculopathy is nerve root compression and can result in numbness in the affected extremity. 5. Radiculopathy is nerve root compression and can result in the inability to control motor movement in the affected area.

A patient has a new onset of back pain radiating down the left leg. For which health problem should the nurse expect this patient to be evaluated? 1) Meningitis 2) Spinal cord tumor 3) Multiple sclerosis 4) Amyotrophic lateral sclerosis

ANS: 2 1 Back pain radiating down the leg is not a manifestation of meningitis. 2 Many of the symptoms of a spinal cord tumor are the same as a low back injury or herniated disk. 3 Although spinal cord tumors can mimic the manifestations of multiple sclerosis, the patient's symptoms are limited to low back pain radiating down the leg. Multiple sclerosis has many additional manifestations which this patient is not experiencing. 4 Manifestations of back pain radiating down the leg are not associated with amyotrophic lateral sclerosis.

A patient with multiple sclerosis is developing speech difficulties. What should the nurse realize as being the reason for this new manifestation? 1) Depression 2) Medications 3) Nerve regeneration 4) Mental status changes

ANS: 2 1 Depression is an adverse effect of the disease. It does not cause speech deficits. 2 Speech defects due to muscle weakness may be due to medications. 3 Nerve regeneration would improve speech. 4 Mental status changes is an adverse effect of the disease. It does not cause speech deficits.

11. The nurse is preparing materials for the families of patients who have sustained a stroke. What information should the nurse include to reduce the risk for additional strokes? 1) Heart-healthy diet 2) Smoking cessation 3) Stress management 4) Weight-reduction strategies

ANS: 2 Feedback 1 A heart-healthy diet might be appropriate if the stroke is caused by atherosclerosis. 2 Smoking cessation is essential for the nurse to include. Exposure to nicotine due to cigarette smoking causes a decrease in oxygen levels in the blood, which may contribute to blood clot formation and vasoconstriction with each inhalation of smoke. Additionally, nicotine may cause more rapid deposition and accumulation of atherosclerotic plaque. Patients may be more willing to consider smoking cessation after an illness such as stroke. 3 Stress management has not been identified as essential teaching to reduce the risk of stroke. 4 Weight reduction has not been identified as essential teaching to reduce the risk of stroke.

9. A patient recovering from a hemorrhagic stroke has a blood pressure of 90/50 mm Hg. What action should the nurse take? 1) Increase the head of the bed 2) Notify the health-care provider 3) Place the head in a neutral position 4) Reassess the pressure in 15 minutes

ANS: 2 Feedback 1 Increasing the head of the bed could further reduce blood flow to the brain. 2 A protective mechanism of the brain, cerebral autoregulation, is dysfunctional after stroke, rendering the brain vulnerable to hypotension because the cerebral blood vessels are not able to automatically dilate ensuring adequate oxygen delivery to brain tissue. The health-care provider should be notified with the blood pressure measurement. 3 Placing the head in a neutral position will not impact the patient's blood pressure. 4 The patient's condition could deteriorate in 15 minutes. The blood pressure should be reported immediately.

2. A patient with increased intracranial pressure (ICP) is sensitive to fluid-volume shifts. Which approach would be the safest to reduce this patient's cerebral edema? 1) Mannitol 2) 3% normal saline 3) Bacteriostatic saline 4) Preservative-free saline

ANS: 2 Feedback 1 Mannitol pulls water from the interstitial spaces across the blood-brain barrier into the vascular space for diuresis through the kidney. This medication can cause dramatic fluid shifts and should be provided with intravenous fluids to prevent reactions to fluid imbalances. 2 High-concentration sodium chloride solutions pull water from the interstitial spaces into the vascular space without the dramatic fluid shifts caused when osmotic diuretics are utilized 3 Bacteriostatic saline is used to flush the intracranial pressure monitoring device. 4 Preservative-free saline solution is used to flush the intracranial pressure monitoring device.

4. A patient with increased intracranial pressure (ICP) has a body temperature of 100°F. What action should the nurse take to address this temperature elevation? 1) Place head in a neutral position 2) Administer antipyretic as prescribed 3) Auscultate lung sounds and increase fluids 4) Send a urine sample for culture and sensitivity

ANS: 2 Feedback 1 Placing the head in a neutral position will not affect body temperature. 2 Antipyretics should be administered for temperature elevations because it will prevent an increase in cerebral metabolism. 3 Auscultating lung sounds and increasing fluids assumes the temperature is caused by a lung infection or pooling of secretions. This is not an identified action for a temperature elevation. 4 Sending a urine specimen for analysis assumes that the temperature elevation is caused by a urinary tract infection. This is not an identified action for a temperature elevation.

1. The nurse is preparing a patient for insertion of an intraventricular catheter intracranial pressure (ICP) monitoring device. What is an advantage of this device? 1) Must be inserted in the operating room 2) Catheter tip located in the lateral ventricle 3) Less mechanical drift of the measurement over time 4) Lower rate of infection because of no fluid reservoir

ANS: 2 Feedback 1 The intraventricular catheter ICP monitoring device can be inserted at the bedside or the operating room. This is not an advantage. 2 The intraventricular catheter ICP monitoring device is considered the gold standard for ICP measurement because the catheter tip is located in the lateral ventricle. 3 The intraparenchymal sensor/probe has less mechanical drift of the measurement over time. 4 The subarachnoid bolt (SAB) has a lower rate of infection because of no fluid reservoir.

The nurse is concerned that a patient is at risk for a lumbar disk herniation. What assessment information is causing the nurse to have this concern? Select all that apply. 1) Fell off of a ladder 2) Body mass index 32.3 3) Smokes 1 pack of cigarettes per day 4) Works in an automotive factory 5) Lifts 50 lb. sack of cement at work

ANS: 2, 3, 4, 5 1. A traumatic event such as a fall or a blow to the back rarely causes a herniated disk. 2. Factors that increase the risk of a herniated disk include obesity. 3. Factors that increase the risk of a herniated disk include smoking. 4. Factors that increase the risk of a herniated disk include occupation that includes repetitive lifting, pulling, pushing, bending sideways, and twisting. 5. Factors that increase the risk of a herniated disk include occupation that includes repetitive lifting, pulling, pushing, bending sideways, and twisting.

The nurse is assessing a patient with multiple sclerosis. What should the nurse expect to assess in this patient? Select all that apply. 1) Anxiety 2) Dizziness 3) Double vision 4) Unsteady gait

ANS: 2, 3, 4, 5 1. Anxiety is not an identified manifestation of multiple sclerosis. 2. Depending on the location of the affected nerve fibers, a manifestation of multiple sclerosis includes dizziness. 3. Depending on the location of the affected nerve fibers, a manifestation of multiple sclerosis includes double vision. 4. Depending on the location of the affected nerve fibers, a manifestation of multiple sclerosis includes unsteady gait. 5. Depending on the location of the affected nerve fibers, a manifestation of multiple sclerosis includes electric shocks with head movement.

27. A patient is being discharged after treatment for an ischemic stroke. Which medications should the nurse expect to be prescribed for this patient? Select all that apply. 1) Antibiotics 2) Anticoagulant 3) Antihypertensive 4) Antiplatelet therapy 5) Lipid-lowering agent

ANS: 2, 3, 4, 5 Feedback 1. Antibiotics are not routinely prescribed in the treatment of an ischemic stroke. 2. According to primary stroke center accreditation guidelines, stroke patients should be discharged with anticoagulation if indicated for atrial fibrillation. 3. According to primary stroke center accreditation guidelines, stroke patients should be discharged with a blood pressure control strategy in patients with hypertension. 4. According to primary stroke center accreditation guidelines, stroke patients should be discharged with antiplatelet therapy. 5. According to primary stroke center accreditation guidelines, stroke patients should be discharged with lipid-lowering therapy if indicated.

The nurse is reviewing orders written for a patient returning from surgery to remove a spinal cord tumor. Which order should the nurse implement immediately? 1) Install a bed cradle 2) Apply heel protection 3) Dexamethasone (Decadron) 10 mg IVP now and every 8 hours x 2 doses 4) Monitor bowel sounds; notify if unable to obtain or faint in any abdominal quadrant

ANS: 3 1 A bed cradle can be installed at any time. 2 Heel protection can be applied at any time. 3 Corticosteroid therapy is prescribed after surgery to decrease swelling and inflammation. This is the priority. 4 Monitoring of bowel sounds can occur after the medication is provided.

A patient is admitted for diagnosis and treatment of ongoing spasticity and flaccidity of the extremities. Which diagnostic test should the nurse expect to be prescribed that will definitively determine this patient's health problem? 1) MRI of the neck 2) CT scan of the head 3) A variety of tests will be ordered to help rule out other causes 4) Analysis of cerebrospinal fluid from a lumbar puncture

ANS: 3 1 An MRI of the neck may be completed to rule out nerve compression. 2 A CT scan of the head may be completed to rule out masses or other structural abnormalities causing the patient's symptoms. 3 No single test can be used to diagnose ALS; therefore, a complete history and physical examination must be performed by the health-care provider. 4 Analysis of the cerebrospinal fluid will be done; however, this will not definitively diagnose the patient with ALS.

A patient is diagnosed with spinal stenosis. Which manifestation is caused by age-related loss of spinal muscle strength? 1) Fatigue 2) Constipation 3) Muscle spasm 4) Decreased stamina

ANS: 3 1 Fatigue could occur because of needing to be sedentary to prevent the onset of pain. 2 Constipation could occur because of decreased mobility. 3 Muscle spasm is the sudden, involuntary contraction of a single muscle or muscle group. Most back muscle spasms are due to inflammation and soreness because of sudden movement or bending, but they can also result from age-related loss of spinal muscle strength as muscles spasm to protect the worn areas of the spine. 4 Decreased stamina is possible if pain decreases activity and the patient becomes more sedentary.

A patient with low back pain asks what aspirin is supposed to do help with the pain. How should the nurse respond to this patient? 1) "It depresses the central nervous system." 2) "It blocks sodium channels and stops the formation of nerve impulses." 3) "It blocks enzymes and chemicals in the body to decrease pain and inflammation." 4) "It blocks the production of substances that trigger allergic and inflammatory reactions."

ANS: 3 1 Muscle relaxants depress the central nervous system. 2 Tricyclic antidepressants block the sodium channels and decrease formation of ectopic neuronal pacemakers. 3 NSAIDs block enzymes and prostaglandins throughout the body, thereby decreasing pain and inflammation. 4 Corticosteroids block the production of substances that trigger allergic and inflammatory reactions.

15. The nurse is caring for a patient with a diffuse axonal injury. What treatment plan should the nurse expect to be prescribed for this patient? 1) Craniotomy 2) Wound debridement 3) Monitor and observe 4) Evacuation of the hematoma

ANS: 3 Feedback 1 A craniotomy may be indicated in patients with hemorrhagic injuries and typically involve opening of the skull and removal of blood accumulations. 2 With skull fractures, there is a need to débride and clean the wound area as well as to remove any bone fragments that may be at the area of impact. 3 Surgery is not indicated in patients with diffuse axonal injuries because there is no specific area of blood removal. 4 Both epidural and subdural hematomas are surgically evacuated to decrease ICP.

13. The nurse is assessing a patient who sustained a traumatic brain injury several years ago. What finding should the nurse expect when completing the assessment? 1) Dysphagia 2) Hemiparesis 3) Memory loss 4) Visual field deficits

ANS: 3 Feedback 1 Dysphagia is associated with a stroke. 2 Hemiparesis is associated with a stroke. 3 Many who sustain even a mild head injury suffer long-term effects such as memory loss 4 Visual field deficits are associated with a stroke.

14. An adolescent seeks medical care after being in a street fight. Which observation indicates that this patient has sustained a basilar skull fracture? 1) Hyperthermia 2) Episodic tachycardia 3) Bruising around the ears 4) Rapid deterioration to comatose

ANS: 3 Feedback 1 Hyperthermia is a manifestation of a diffuse axonal injury. 2 Episodic tachycardia is a manifestation of a diffuse axonal injury. 3 A late sign of a basilar fracture is bruising around the eyes (raccoon's eyes) or the ears, termed Battle's sign. 4 Rapid deterioration to comatose is a manifestation of an epidural hematoma.

7. The nurse suspects that a patient is experiencing a hemorrhagic stroke from a ruptured cerebral aneurysm. What assessment finding caused the nurse to make this conclusion? 1) Slurred speech 2) Visual field deficits 3) Sudden severe headache 4) Lower extremity weakness

ANS: 3 Feedback 1 Slurred speech is not identified as a manifestation of a ruptured cerebral aneurysm. 2 Visual field deficits are not identified as a manifestation of a ruptured cerebral aneurysm. 3 Subarachnoid hemorrhage is characterized by a sudden severe headache, often termed a "thunderclap" headache because of the intensity of the pain experienced at the onset. 4 Lower extremity weakness is not identified as a manifestation of a ruptured cerebral aneurysm.

19. The nurse is caring for a patient with neurogenic shock. What finding should the nurse expect to assess in this patient? 1) Tachycardia 2) Hypertension 3) Warm dry skin 4) Rapid shallow respirations

ANS: 3 Feedback 1 The clinical presentation of neurogenic shock includes profound bradycardia. 2 The clinical presentation of neurogenic shock includes hypotension. 3 Patients with neurogenic shock typically have warm and dry skin due to systemic vasodilation. 4 The clinical presentation of neurogenic shock includes metabolic acidosis. Rapid shallow respirations would be seen in respiratory alkalosis.

3. The nurse is caring for a patient in a barbiturate coma for increased intracranial pressure (ICP). What should the nurse assess to determine this patient's cerebral function? 1) Gag reflex 2) Glasgow coma scale 3) Pupillary size and reaction 4) Blood pressure and heart rate

ANS: 3 Feedback 1 The patient in a barbiturate coma will not have a gag reflex. 2 The Glasgow coma scale would be inconclusive since the patient is in a coma. 3 The ability to elicit a neurological assessment from a patient in a barbiturate coma is limited because of the suppression caused by the medication; therefore, other parameters such as pupillary size and reaction must be used. 4 Blood pressure and heart rate will not provide maximum information about the patient's cerebral functioning.

A patient asks what smoking cigarettes has to do with low back pain. How should the nurse respond to this patient? 1) "Smoking is a sedentary activity." 2) "Smoking is linked to nutritional disorders." 3) "Nicotine in cigarettes interferes with nutrients that supply the disk spaces." 4) "Nicotine hinders the mobility of the vertebral spaces and interferes with nerve function."

ANS: 3. 1 There is no evidence to link smoking with a sedentary lifestyle. 2 There is no evidence to link smoking with nutritional disorders. 3 Nicotine in cigarettes is thought to interfere with vital nutrients being absorbed by the intervertebral disks. 4 There is no evidence to link nicotine to vertebral space mobility and nerve function.

The nurse is reviewing orders written for a patient with a new spinal cord injury. Which order should the nurse question before completing? 1) Place on air mattress 2) Insert a nasogastric tube and attach to low suction 3) Insert indwelling urinary catheter; strict intake and output 4) Dexamethasone (Decadron) 10 mg IVP now and repeat in 4 hours

ANS: 4 A patient with a spinal cord injury is at risk for skin breakdown and pressure ulcers. An air mattress would be appropriate. 2 A patient with a spinal cord injury is at risk for paralytic ileus, septic or necrotic bowel, and a GI bleed. A nasogastric tube to suction would be appropriate. 3 A patient with a spinal cord injury is at risk for urinary incontinence, urinary tract infection, neurogenic bladder, and chronic kidney disease. An indwelling urinary catheter and strict intake and output measurement would be appropriate. 4 Current guidelines for the management of acute SCIs no longer recommend the use of corticosteroids for acute SCI. Because there is no strong medical evidence supporting benefits from the administration of corticosteroids, this therapy is avoided because of stronger evidence that the administration of high-dose steroids is associated with harmful side effects, including hyperglycemia and immunosuppression.

The latest blood pressure reading for a patient with a spinal cord injury is 210/140 mm Hg. What action should the nurse take first? 1) Palpate for bladder distention 2) Assess for a bowel impaction 3) Re-measure the blood pressure 4) Raise the head of the bed 45 degrees

ANS: 4 1 After the head of the bed is raised the nurse can begin to assess for the cause of autonomic dysreflexia, which includes palpating the bladder. 2 After the head of the bed is raised the nurse can begin to assess for the cause of autonomic dysreflexia, which includes assessing for a bowel impaction. 3 Re-measuring the blood pressure will take precious time that is better spent beginning interventions. 4 The head of the bed should be raised first so that blood can pool in the lower extremities and help reduce the blood pressure.

A patient receives a definitive diagnosis of multiple sclerosis. What finding occurred to validate this diagnosis? 1) Onset of double vision 2) Loss of bowel and bladder control 3) Numbness and tingling of one limb 4) MRI changes in two separate locations

ANS: 4 1 Double vision is a manifestation of multiple sclerosis; however, it does not provide a definitive diagnosis of the disease. 2 Loss of bowel and bladder control is a manifestation of a herniated disk. 3 Numbness and tingling of one limb is a manifestation of multiple sclerosis; however, it does not provide a definitive diagnosis of the disease. 4 For a definitive diagnosis multiple sclerosis, the patient must have MRI changes in at least two separate locations.

The nurse is caring for a patient with multiple sclerosis. What should the nurse do to increase venous return, prevent stiffness, and maintain muscle strength and endurance? 1) Administer interferon 2) Administer corticosteroids 3) Turn and reposition every two hours 4) Encourage range-of-motion exercises

ANS: 4 1 Interferon decreases exacerbations and slows disease progression. 2 Corticosteroids decreases the inflammatory processes associated with the flare. 3 Turning and repositioning every two hours prevents skin breakdown. 4 Range-of-motion exercises increases venous return, prevents stiffness, and maintains muscle strength and endurance.

A client with amyotrophic lateral sclerosis is prescribed riluzole (Rilutek). What statement indicates additional teaching is required about the effects of this medication? 1) "This will cure my disease." 2) "This will help me stay awake." 3) "This will stop my bladder spasms." 4) "This will increase the progression of my disease."

ANS: 4 1 Riluzole (Rilutek) does not repair damaged neurons but has been shown both to increase survival and to extend the period without the need for ventilator support. 2 Analeptics improve wakefulness. 3 Antispasmodics improve bladder spasms. 4 Riluzole (Rilutek) is the first drug approved to slow disease progression.

The nurse is caring for a patient with unrelenting low back pain caused by a herniated disk. What instruction should the nurse provide to this patient to help with the pain? 1) Sit with the legs elevated 2) Reduce the intake of fluids 3) Limit activity until the pain subsides 4) Bend at the knees with a straight back

ANS: 4 1 Sitting with the legs elevated puts pressure on the lower spine. 2 Reducing the intake of fluids can cause constipation, which worsens back pain. 3 Activity should be performed as prescribed to build core muscle strength and stabilize and support the spine. 4 Bending at the knees with the back straight uses the leg muscles to lift and decreases strain on the back muscles.

A patient is experiencing increasing flaccid upper arms while the lower extremities periodically cramp and contract. On which health problem should the nurse focus when assessing this patient? 1) Brain tumor 2) Spinal cord tumor 3) Multiple sclerosis 4) Amyotrophic lateral sclerosis (ALS)

ANS: 4 1 The manifestations of a brain tumor will depend upon the location of the mass in the cerebrum. 2 The manifestations of a spinal cord tumor will depend upon the location of the mass within the cord. 3 Both spasticity and flaccidity do not need to be present to diagnose multiple sclerosis. 4 To be diagnosed with ALS, patients must have clinical manifestations of both upper and lower motor neuron damage that cannot be attributed to other causes. Upper motor neuron damage is associated with spasticity, while lower motor neuron damage is characterized by flaccidity.

18. A patient recovering from a stroke has profound bradycardia. What should the nurse suspect as the cause of this manifestation? 1) Parasympathetic nervous system disruption 2) Irritation of the sympathetic nervous system 3) Shunting of fluid from the cerebral vasculature 4) Alteration in the vasomotor center in the brainstem

ANS: 4 Feedback 1 Neurogenic shock is caused by a disruption in the sympathetic nervous system. 2 Neurogenic shock is caused by a disruption in the sympathetic nervous system. 3 Neurogenic shock is not caused by the shunting of fluid from the cerebral vasculature. 4 When neurogenic shock is caused by stroke in the brainstem, symptoms of neurogenic shock arise from the vasomotor center in the brainstem.

12. A patient recovering from an ischemic stroke is prescribed verapamil (Calan). In preparation for patient teaching, which medication category will the nurse review? 1) Diuretic 2) Beta blocker 3) Lipid-lowering agent 4) Calcium channel blocker

ANS: 4 Verapamil (Calan) is a calcium channel blocker.

A patient with multiple sclerosis experiences exacerbations of new symptoms that last a few days and then disappear. Which type of multiple sclerosis is this patient most likely experiencing? 1) Relapsing-remitting 2) Primary progressive 3) Progressive relapsing 4) Secondary progressive

ANS:1 1 In relapsing-remitting multiple sclerosis, relapses or exacerbations occur during which new symptoms appear and old ones worsen or reappear; these relapses can last days or months. 2 Primary progressive multiple sclerosis has gradual progression with no remissions. 3 Progressive relapsing multiple sclerosis has a gradual worsening of symptoms from onset, and the relapses may or may not have recovery. 4 Secondary progressive multiple sclerosis is when the patient initially had relapsing-remitting but it gradually becomes worse.

A patient being treated with chemotherapy for cancer complains of fatigue, pallor, progressive weakness, exertional dyspnea, headache, and tachycardia. Which diagnosis should the nurse use as the priority when planning this patient's care?

Activity Intolerance

The nurse is providing care to a patient who is intubated and receiving mechanical ventilation after a motor vehicle crash. The patient is fighting the ventilator and attempting to remove the endotracheal tube. Which nursing action decreases the patient's risk for developing respiratory alkalosis?

Administer a prescribed sedative. -for a pt mechanically ventilated, the only way to reduce rapid respirations might be to provide a sedative.

The nurse is caring for a patient with a history of latex allergies. The patient develops audible wheezing, pruritus, urticaria, and signs of angioedema. Which is the priority intervention for this patient?

Administer epinephrine 1:1,000 by subcutaneous injection per the health-care provider's orders.

The nurse is reviewing new orders provided by the health-care provider for a critical care patient with *metabolic acidosis*. Which prescription should the nurse question?

Administer one ampule of sodium bicarbonate now. -administering bicarbonate to correct acidosis increases the risk for *hypernatremia, hyperosmolarity & fluid volume excess*

The nurse is preparing a seminar that discusses the risk and incidence of cancer and culture. What information is considered culturally correct when teaching about the risk of developing cancer?

African-Americans are more likely to develop cancer than any other ethnic group

Which patient should the nurse offer the opportunity for human immunodeficiency virus (HIV) testing during an annual physical examination?

An 18-year-old young adult patient

A patient has just been told that a colectomy and ileostomy are needed to treat a new diagnosis of colon cancer. Which nursing diagnosis should the nurse use to plan this patient's preoperative nursing care?

Anticipatory Grieving

Hypocalcemia

Anxiety, confusion, irritability, paresthesias, *+ Chvostek's/Trousseau* sign, tetany, twitching, tremors

The nurse is providing care to a patient with autoimmune hepatitis. Which medication should the nurse prepare to teach this patient about based on the diagnosis?

Azathioprine

The nurse is caring for a patient with leukemia. Which treatment should the nurse expect to be prescribed?

Chemotherapy

The nurse is providing care to a patient with psoriasis. Which medication should the nurse prepare to teach this patient about based on the diagnosis?

Cyclosporine

A patient receiving radiation therapy as treatment for colorectal cancer is experiencing nausea and vomiting. What should the nurse encourage the patient to do?

Delay the intake of a meal until three to four hours after treatment

The nurse is providing care to a patient who is diagnosed with (HIV). The patient has lost 15% of body weight since the last appointment. Which reason should the nurse include in a teaching session for this patient regarding this occurrence?

Disease progression

Hypernatremia

Disorientation, hallucinations, agitation, restlessness, confusion, seizures, lethargy, tachycardia, dry mucous membranes, skin flushed, agitation, thirst

The nurse is reviewing the health-care provider orders for a patient who is diagnosed with *respiratory alkalosis*. Which prescription is appropriate for this patient's care needs?

Draw arterial blood gas analysis. -management of respiratory alkalosis focuses on correcting & treating the underlying cause. ABGs must be ordered prior to beginning meds or oxygen therapy

The nurse is caring for a patient with acquired immunodeficiency syndrome (AIDS) who is in antiretroviral therapy. The patient reports nausea, fever, severe diarrhea, and anorexia. Which prescribed medication does the nurse anticipate in order to relieve the anorexia and to stimulate the patient's appetite?

Dronabinol (Marinol)

Which should the nurse include in the plan of care for a patient diagnosed with DiGeorge's syndrome to treat gastrointestinal reflux disorder (GERD)?

Droplet precautions

Hypermagnesemia

ECG changes, widened QRS, hypotension, bradycardia, drowsiness, lethargy, decreased deep tendon reflexes

Which intervention should the nurse implement for a patient whose *serum phosphorus level is 2.0 mg/dL*?

Encourage consumption of milk and yogurt A phosphorus level of 2.0 is low, and the client will need additional dietary phosphorus. Providing phosphorus-rich foods such as milk and yogurt is a good way to provide that additional phosphorus.

(Box 9.6) Causes of *Metabolic Acidosis*

Excess of acid • Renal failure • Ketoacidosis • Anaerobic metabolism • Starvation • Salicylate intoxication Loss of base • Diarrhea • Intestinal fistulas

Which general manifestation should the nurse anticipate when providing care to a patient diagnosed with DiGeorge's syndrome?

Failure to thrive

Transcellular

GI, respiratory, urinary tracts, glandular, intraocular, cerebrospinal

Which is the priority nursing action to decrease the risk for infection for a patient diagnosed with DiGeorge's syndrome?

Hand hygiene

Hyponatremia

Headache, lethargy, confusion, convulsions, nausea/vomiting, coma

The nurse is providing care to a patient diagnosed with human immunodeficiency virus (HIV). The patient's CD4+ count is currently 480 cells/μL. Which complication is this patient at risk for developing?

Herpes zoster virus

The nurse is caring for a patient who is experiencing anaphylactic shock following the administration of a medication. Which position is the most appropriate for the nurse to place the patient based on this data?

High Fowler position

The nurse is assisting the health-care provider with a bone marrow aspiration and biopsy on a patient who has leukemia. The patient also has thrombocytopenia. Upon completing of the test, which intervention is a priority for the nurse?

Hold pressure on the wound for approximately five minutes

A patient with terminal colon cancer is refusing all food and fluids. The patient has a living will that states no artificial nutrition is to be provided; however, the family is asking for a gastrostomy tube. What should the nurse do?

Honor the patient's refusal and help the family come to terms with the situation.

A nurse is caring for a patient with cancer. The nurse teaches the patient about which potentially undesirable cellular alterations that can occur during the cell cycle?

Hyperplasia

A nurse is caring for a pediatric patient who is receiving an infusion of intravenous antibiotic at the ambulatory clinic. Which clinical manifestation indicates that the patient is experiencing a type I hypersensitivity reaction?

Hypotension

A patient with anemia caused by chemotherapy is prescribed synthetic erythropoietin. When teaching the patient about the therapeutic effect of this treatment, which is appropriate for the nurse to include?

Increase in red blood cells

The nurse correlates which physiological response to the secretion of natriuretic peptide hormones?

Increase in urine output -Natriuretic hormones are released in response to increased BV and BP. As a result of the release of ANP and BNP, reabsorption of sodium by the kidney and increase in glomerular filtration rate results in increased U.O that is high in sodium

The nurse is providing care to a patient who is diagnosed with (HIV). The patient presents with a fever without other notable symptoms. Which is the most likely cause of this data?

Infection

Which immunization should the nurse encourage for a patient who is diagnosed with Stage 2 human immunodeficiency virus?

Influenza vaccine

The nurse is providing care for a patient diagnosed with agammaglobulinemia. Which is the anticipated treatment for this patient?

Intravenous immunoglobulin (IVIG)

A patient is prescribed *20 mEq of potassium chloride due to excessive vomiting*. Which is the rationale for this drug the nurse should provide to the patient?

It is needed to *maintain skeletal, cardiac, and neuromuscular activity*. Potassium is the major cation in intracellular fluids, with only a small amount found in plasma and interstitial fluid. *Potassium is a vital electrolyte for Skeletal, Cardiac, & SM activity*.

A nurse is caring for a patient with seasonal hypersensitivity reactions. What teaching would the nurse provide to improve this patient's comfort?

Maintain a clean, dust-free environment.

Which is the *priority* nursing action when providing care to a patient who is admitted with *metabolic alkalosis*?

Monitoring *oxygen saturation* -the depressed respiratory drive that often accompanies metabolic alkalosis can lead to hypoxemia and impaired oxygenation of the tissues

The nurse is providing care to a pediatric patient who is diagnosed with DiGeorge's syndrome. Which data indicates a cardiovascular abnormality? Select all that apply.

Murmur Cyanosis Failure to thrive Cleft lip and palate

Hyperkalemia

Muscle cramps, paresthesia to weakness, ECG changes (abnormal rhythms, widened QRS complex)

Hyperphosphatemia

Muscle cramps, paresthesia, weakness, decreased DTRs, ECG changes

The nurse is providing care to a patient who is diagnosed with human immunodeficiency virus (HIV). The patient reports night sweats. Which is the most likely reason for this clinical manifestation?

Mycobacterial infection

The nurse is providing care to a patient with lupus. Which medication should the nurse prepare to teach this patient about based on the diagnosis?

Mycophenolate mofetil

The nurse is preparing to assess a patient when one of the patient's family members begins showing symptoms of a latex sensitivity. Which action by the nurse is the most appropriate?

Obtain latex-free products for the patient's room

The nurse is reviewing the latest ABG results for a patient with *metabolic alkalosis*. Which result indicates that the *metabolic alkalosis is compensated*?

PaCO2 48 mmHg

Hypomagnesemia

Paresthesias, tetany, twitching, tachycardia, nausea/vomiting, seizures, anorexia, and dysrhythmias such as rapid heart rate and PVCs, ventricular tachycardia, emotional lability, and deep tendon reflexes increased

The nurse suspects that the patient is experiencing a reaction to a specific antigen. Which laboratory result supports the conclusion made by the nurse?

Patch test with a 1-inch area of erythema

The nurse is caring for an adolescent Asian patient with a strong family history of breast cancer. What should the nurse teach the patient regarding cancer prevention?

Perform monthly breast self-examination

The nurse is providing care to a patient who is suspected of having an immune deficiency. Which information in the patient's health history supports this suspected diagnosis? Select all that apply

Persistent oral thrush Two serious sinus infections in a one-year period

(CC 8.7) Which nursing action is the priority in the patient with a serum calcium of 6.0 mg/dL?

Place an intubation tray at the bedside -in pt w/ HYPOcalcemia, due to potential of laryngeal stridor, cardiac dysrhythmias & seizures, emergency equipment such as intubation equipment, tracheostomy set, code/emergency cart & defibrillator must be ready

A nurse is caring for a patient with leukemia who is neutropenic. Which intervention will the nurse implement to ensure this patient's safety?

Place patient in reverse isolation

The nurse is providing care to a patient who is diagnosed with human immunodeficiency virus (HIV). The patient has shortness of breath when walking, but no problems breathing at rest. Which is the most likely cause for this clinical manifestation?

Pneumocystis carinii pneumonia

*Intra*cellular

Potassium Magnesium Phosporus

The nurse is assessing a patient who is receiving intravenous (IV) antibiotics. Which item in the patient's health history increases the risk for experiencing a hypersensitivity reaction?

Previous antibiotic therapy

The nurse accompanies the health-care provider into the patient's room and listens as the diagnosis of cancer is shared with the patient and family. Once the health-care provider leaves the room, the nurse notes that the patient and family are teary-eyed regarding the diagnosis. What is the nurse's most appropriate intervention at this time?

Provide emotional support in coping with the diagnosis.

The nurse is providing care to patient with the following laboratory values: pH - 7.31; PaCO2 - 48 mmHg; and a normal HCO3. Which condition should the nurse plan care for based on the current data?

Respiratory acidosis -to compensate for this imbalance, the rate and depth of respirations decrease, leading to retention of CO2. PaCO2 will be elevated.

The nurse is caring for a patient with leukemia who is experiencing neutropenia as a result of chemotherapy. Which action should the nurse include in the plan of care for this patient?

Restrict visitors with communicable illnesses

During an assessment, the nurse notes that a patient receiving radiation treatments for breast cancer has excoriated skin. What is the priority nursing diagnosis?

Risk for Infection

The nurse is providing care to a patient diagnosed with human immunodeficiency virus (HIV). The patient's CD4+ count is currently 250 cells/μL. Which complication is this patient at risk for developing?

Severe bacterial infection

Which respiratory manifestation should the nurse anticipate when providing care to a patient diagnosed with DiGeorge's syndrome?

Shortness of breath

During a treatment meeting on an oncology unit, the nurse learns that a patient is scheduled for chemotherapy before surgery. What are the purposes for this patient to receive chemotherapy at this specific time?

Shrink the tumor

A nurse is working in a summer camp for children. One of the children comes to the clinic with several bee stings. Which clinical manifestations would necessitate the need to inject the child with epinephrine (EpiPen)? Select all that apply.

Skin that is cold and clammy to the touch Complaints of thirst Restlessness and confusion

The nurse is providing care to a patient, diagnosed with human immunodeficiency virus (HIV), with a CD4+ count of 300 cells/μL. Which classification of HIV should the nurse document for this patient?

Stage 2

The nurse is providing care to a patient, diagnosed with human immunodeficiency virus (HIV), with a CD4+ count of less than 200 cells/μL. Which classification of HIV should the nurse document for this patient?

Stage 3

A nurse working in an intensive care unit (ICU) is assigned a patient diagnosed with acquired immunodeficiency syndrome (AIDS). Based on this data, which type of precaution does the nurse implement when providing direct care?

Standard

A patient being treated for cancer has a tumor designation of Stage IV, T4, N3, M1. What does this staging indicate to the nurse?

The tumor has metastasized with lymph node involvement

he nurse is providing care to a patient diagnosed with human immunodeficiency virus (HIV). The patient's CD4+ count is less than 200 cells/μL. Which complication is this patient at risk for developing?

Toxoplasmosis

The nurse is providing care to a patient diagnosed with human immunodeficiency virus (HIV). The patient's CD4+ count is greater than 500 cells/μL. Which complication is this patient at risk for developing?

Vaginal candidiasis

The nurse is discharging a pediatric patient who was recently diagnosed with acquired immunodeficiency syndrome (AIDS). When discussing appropriate health promotion activities for this child, which immunization is contraindicated?

Varicella vaccine

Which is the priority action for a nurse who is exposed to a needle-stick injury while providing patient care?

Washing the injury under running water

Hypokalemia

Weakness, lethargy, hyporeflexia, ECG changes (ST depression), PVCs, nausea/vomiting, constipation, abdominal cramping

HYPOvolemia CM's

Weight loss, loss of skin turgor, concentrated urine output, oliguria (low urine output), thirst, and dry mucous membranes are indications of fluid volume deficit additional: weak, rapid peripheral pulses, flattened neck veins, hypotension, anxiety, restlessness, and cool, clammy, pale skin

osmolarity normal range

[*170-300*] mOsm/L

Serum magnesium

[1.6-2.6] mg/dL

Serum sodium

[135-145] mEq/L

Serum phosphorus

[2.5-4.5] mg/dL

Urine osmolality

[250-900] mOsm/kg

Serum osmolality

[275-295] mOsm/kg

Serum potassium

[3.5-5.0] mEq/L

Ionized calcium

[4.6-5.3] mg/dL

Total serum calcium

[8.2-10.2] mg/dL

Serum chloride

[97-107] mEq/L

Hypervolemia causes

cirrhosis, HF, stress conditions causing a release of ADH and aldosterone, adrenal gland disorders, and use of corticosteroids, extra salt

HYPOvolemia causes

excessive loss of fluids, insufficient intake of fluid, or fluid shifts: vomiting, diarrhea, nasogastric succ, increased perspiration, hemorrhage, *diabetes insipidus*, *DKA*, adrenal insufficiency

Crystalloid Isotonic

greater than 250 mOsm/L 1. *0.9%* NS (vascular expansion) 2. Lactated Ringer's (*LR*) (electrolyte replacement) considerations: may cause: -fluid overload -generalized edema -dilutes hemoglobin -*hyperchloremic acidosis* -electrolyte imbalance -proinflammatory in large doses

crystalloid hypertonic

greater than 375 mOsm/L 1. *D5 0.45% NS* 2. *D5 0.9% NS* 3. HS 3% or 5%* considerations (may cause): -irritating to veins -fluid overload -*hyperCatremia* -*hyperChloremia* (HS slows inflammation % increases capillary permeability)

hypervolemia NM

health history, performing a physical assessment, reviewing laboratory data, weight, and calculation of I&O Physical assessment: -adventitious breath sounds (crackles) -the presence of the extra heart sound S3, -abdominal distention or ascites -peripheral edema -a distended jugular vein -altered mental status.

HYPOvolemia NM

health history, physical assessment, review of diagnostic results, and calculation of I&O LABs may include elevated hemoglobin & hematocrit, serum sodium levels, & BUN

administer hypertonic saline solution (3% or 5% NaCl)

in pt's exhibiting neurological symptoms

HYPOvolemia LABs

include -electrolytes, specifically sodium and potassium (hypokalemia) -decreased hemoglobin & hematocrit -serum osmolality -elevation of BUN to creatinine ratio -urine specific gravity, & urine osmolality

Euvolemic hyponatremia

increase in total body water w/ no evidence of edema or hypovolemia. seen in SIADH & other endocrine disorders like hypothyroidism or adrenal insuff.

HYPO-volemia medical management

intake of oral fluids is preferred -IV isotonic solutions such as 0.9% normal saline (NS) or lactated Ringer's solution expands plasma volume and corrects hypotension -IV solutions of D5 0.45% NS or 0.45% NS can replace total body water deficits and are used as maintenance fluids

Crystalloid Hypotonic

less than 250 mOsm/L 1. *0.25%* NS 2. *0.45%* NS 3. *0.5%* D5W considerations: -may worsen HTPO-TN -can increase edema (D5Q may irritate veins)

The nurse is providing care to a patient who is admitted to the hospital with *sudden, severe abdominal pain*. Which arterial blood gas supports the patient's current diagnosis of respiratory alkalosis?

pH is 7.47 and PaCO2 is 25. -acute pain causes hyperventilation, causes CO2 to drop and the client to have respiratory alkalosis. The pH would denote alkalosis & be higher than 7.45. HCO3 would trend downward as the kidneys begin to compensate for the alkalosis by secreting HCO3. The PaO2 is likely to be normal unless the pt has been hyperventilating fo ra long time and is beginning to tire.

Safety Alert! *Postural HYPO-TN risk*

pt w/ significant *hypovolemia* -nurse must monitor pt closely when changing the pt's position, particularly when sitting or standing, as the pt is at risk for dizziness and falls

hypervolemia LABs

serum electrolytes, hematocrit, low BUN, serum osmolality, and albumin -Hypoalbuminemia in liver dys-f(x) -Hyponatremia and low serum osmolality may be present in chronic renal failure b/c of diluted BV

hypervolemia CM's

weight gain, ascites, edema, and increased urinary output. cardiac CM: HTN, tachycardia, elevated CVP, development of S3 heart sounds, and jugular vein distention


Related study sets

ATI SM 3.0: Enteral Tube Feeding

View Set

Volume of cubes and rectangular prisms

View Set

Chap 39 Assesment of musculoskeletal function

View Set

The real final exam of accounting for mutlpkle choices

View Set

Women's Health/Disorders and Childbearing Health Promotion

View Set